Top Banner
Free . Regular . Quality PRELIMS TEST SERIES - 2020 Test - 28 www.YouTube.com/SleepyClasses Full-lenght Test III
66

PRELIMS TEST SERIES - 2020 - Sleepy Classes

Mar 29, 2023

Download

Documents

Khang Minh
Welcome message from author
This document is posted to help you gain knowledge. Please leave a comment to let me know what you think about it! Share it to your friends and learn new things together.
Transcript
Page 1: PRELIMS TEST SERIES - 2020 - Sleepy Classes

Free . Regular . Quality

PRELIMS TEST

SERIES - 2020

Test - 28

www.YouTube.com/SleepyClasses

Full-lenght Test III

Page 2: PRELIMS TEST SERIES - 2020 - Sleepy Classes
Page 3: PRELIMS TEST SERIES - 2020 - Sleepy Classes

Free . Regular . Quality

www.courses.sleepyclasses.com

Test No._______________ Topic: ________________________________________

Syllabus: ____________________________________________________________

____________________________________________________________________

Scan QR Codeto download the

Android App.

Page 4: PRELIMS TEST SERIES - 2020 - Sleepy Classes

www.YouTube.com/SleepyClasses

www.sleepyclasses.com/

Full Length III

1. Why Net Domestic Product (NDP) is

sometimes considered as a realistic assessment of an economy’s resources than a Gross Domestic Product (GDP).

Consider the following statements:

1. NDP takes into account taxes and subsidies paid which GDP does not.

2. NDP accounts for economic depreciation which GDP does not.

3. NDP counts the Net income earned by all the Indians living anywhere in the world.

Which of the above is/are correct?

A. 1 and 3 only

B. 2 only

C. 1 and 2 only

D. Neither 1 nor 2

2. In which of the following circumstances the GDP per capita of a country will necessarily increase?

1. Gross capital formation has increased

2. Export goods have increased

3. Index of industrial production has increased

4. Higher remittances have been received in the previous year

Select the correct answer using the codes below.

A. 1 and 2 only

B. 1, 2, and 3 only

C. 2, 3, and 4 only

D. None of the above

3. ASPIRE is one of the main schemes of government of India, Consider the following statements about the same:

1. The scheme is promoted by the Ministry of Skill Development.

2. The scheme aims at setting up Livelihood Business Incubators (LBIs) only.

3. It will be implemented in both Rural and Urban areas.

Which of the statements given above is/are incorrect?

A. 1 and 2 only

B. 2 and 3 only

C. All of the above

D. None of the above

4. Consider the following statements:

1. Shanta Kumar headed the High-Level

Committee on the restructuring of the Food Corporation of India (FCI).

2. The central government announces the Minimum Support Price (MSP) based on the recommendations of the

Cabinet Committee on Economic Affairs.

3. MSP is decided before the harvesting season for crops.

Which of the statements given above is/are correct?

A. 1 and 3 only

B. 1 only

C. 2 and 3 only

D. None of the above

Page 5: PRELIMS TEST SERIES - 2020 - Sleepy Classes

www.YouTube.com/SleepyClasses

www.sleepyclasses.com/

5. G20 is a very important institution in today’s world politics, Consider the following statements about the same:

1. It is an organization that consists of Head of the governments and central bank governors of member countries.

2. The formation of it was conceived after the Asian Financial Crisis in 1997- 1998.

3. The G20 has representation from 19 countries and the European Union only.

Which of the statements given above is/are incorrect?

A. 1 only

B. 2 and 3 only

C. 1 and 3 only

D. 1 and 2 only

6. Consider the following statements about WTO’s different Boxes:

1. Green box subsidies are subject to de minimis limits.

2. Blue box subsidies do not distort trade, or at most cause minimal distortion.

3. Amber box contains any support that would cause trade and production distortions and the support that requires farmers to limit production.

Which of the statements given above is/are correct?

A. 1 and 3 only

B. 2 and 3 only

C. 1 and 2 only

D. None of the above

7. Consider the following statements about National Food Security Act (NFSA):

1. The Act gives legal entitlement to around 75% of urban and 50% of the rural population of India to receive highly subsidized food grains.

2. The foodgrains are allocated at the rate of 5 kg per person per month for priority household’s category and 35 kg per family per month for Antyodaya Anna Yojana families.

3. Highly subsidized prices of Rs 1, 2, and 3 per kg for nutritious-cereals, wheat, and rice respectively are charged.

Which of the statements given above is/are correct with respect to the National Food Security Act (NFSA)?

A. 1 and 2 only

B. 2 and 3 only

C. 1 and 3 only

D. 1, 2 and 3

8. Consider the following statements about Gender Budgeting?

1. It uses a budget as an entry point to apply a gender lens to the entire policy process.

2. It provides for a separate budget for women.

3. The Ministry of Finance is the nodal agency for its implementation.

Select the incorrect answer using the code given below:

A. 1 only

B. 1 and 2 only

C. 2 and 3 only

D. 1 and 3 only

9. With reference to the Equity Grant

Scheme, consider the following statements:

Page 6: PRELIMS TEST SERIES - 2020 - Sleepy Classes

www.YouTube.com/SleepyClasses

www.sleepyclasses.com/

1. Under this scheme, the government grants equity to individual investors Farmer producer organizations

2. This scheme is implemented by NABARD.

Which of the above statements is/are correct?

A. 1 only

B. 2 only

C. Both 1 and 2

D. Neither 1 nor 2

10. Consider the following statements about Money Market instruments:

1. The money market is a market for short term funds whose period of maturity is up to one year.

2. Treasury bills are the short term equity instruments issued by the Government of India.

3. Commercial Paper (CP) is a secured money market instrument issued in the form of a promissory note.

Which of the statements given above is/are correct?

A. 1 and 2 only

B. 1 and 3 only

C. 1 only

D. 1, 2 and 3

11. Consider the following statements with respect to the Infrastructure Investment Funds (InvITs):

1. InvITs pool small sums of money from many investors to invest in assets that give long term returns.

2. InvITs are suitable for high net worth individuals.

3. These funds are Alternate investment funds that are not registered with SEBI.

Which of the statements given above is/are correct?

A. 2 and 3 only

B. 3 only

C. 1 and 2 only

D. 1, 2 and 3 only

12. Utkarsh 2022 scheme of Government of India is related to:

A. Roadmap to make India a 5 trillion economy

B. Merging of Public sector banks

C. Checking the liquidity problem of NBFC’s

D. Improve the regulation and supervision of the central bank

13. Consider the following statements about Non-Banking Financial Companies (NBFCs):

1. NBFCs do not form part of the payment and settlement system and cannot issue cheques drawn on itself

2. NBFCs whose asset size is of Rs. 1000 cr or more as per the last audited balance sheet are considered as systemically important NBFCs.

3. NBFCs can accept time and demand deposits.

Which of the statements given above is/are correct?

A. 1 only

B. 1 and 3 only

C. 2 and 3 only

D. 1, 2 and 3

14. Consider the following statements

Page 7: PRELIMS TEST SERIES - 2020 - Sleepy Classes

www.YouTube.com/SleepyClasses

www.sleepyclasses.com/

1. It mainly involves permanent work contracts.

2. It provides cheaper and more efficient services to the users.

Which of the above are incorrect about the Gig economy?

A. 1 only

B. 2 only

C. Both 1 and 2

D. Neither 1 nor 2

15. Plastic parks are being set up at various locations in India. Consider the following statements about them:

1. It is a zone where used plastic materials are dumped.

2. Odisha has become the first state to start working on developing a Plastic Park.

Which of the statements given above is/are correct?

A. 1 only

B. 2 only

C. 1 and 2 only

D. Neither 1 nor 2

16. Pigovian tax is best described by the term:

A. It is imposed on the profits of pig rearing enterprises.

B. It is imposed on the transactions having negative externalities.

C. It is a tax on all foreign exchange transactions.

D. It is a safeguarding tax against the foreign items being dumped in the country.

17. Consider the following statements :

1. The IMF’s primary purpose is to ensure the stability of the international monetary system.

2. IMF members cannot voluntarily exchange Special Drawing Rights (SDRs) for currencies among themselves.

3. Though India is a member of the IMF, it is not a founding member of it.

4. India has not taken any financial assistance from the IMF since 1993.

Which of the above statements regarding the International Monetary Fund (IMF) are correct?

A. 1, 2 and 3 only

B. 1, 3 and 4 only

C. 1 and 4 only

D. 1 and 3 only

18. Consider the following statements with reference to the Debt to GDP ratio.

1. Debt to GDP ratio of states has increased in the recent Financial Years.

2. The N.K. Singh Committee report had recommended the ratio to be 40% for the States and 20% for the Centre, respectively, by 2023.

Which of the statements given above is/are correct?

A. 1 only

B. 2 only

C. Both 1 and 2

D. Neither 1 nor 2

19. The Concurrent List of the 7th schedule of the constitution contains:

1. Forest

2. Wild animals and birds

3. Administration of justice

Page 8: PRELIMS TEST SERIES - 2020 - Sleepy Classes

www.YouTube.com/SleepyClasses

www.sleepyclasses.com/

4. Education

5. Public order

Select the correct answer using the code given below:

A. 1, 3 and 5 only

B. 1, 2 and 3 only

C. 1, 2,3 and 4 only

D. 1, 2, 3, 4 and 4 only

20. Consider the following statements:

1. The Commission for SCs and STs were bifurcated into two separate bodies by the 89th Constitutional Amendment

Act.

2. Chairperson and members of the National Commission for SCs hold office for a term of five years.

3. They are appointed by a committee consists of P.M, leader of opposition in L.S and Speaker of L.S

Which of the statements given above is/are incorrect?

A. 1 and 2 only

B. 1 only

C. 2 and 3 only

D. None of the above

21. In which of the following matters the power of Legislative Council of a state is not equal to Legislative Assembly :

1. Voting on the demands for grants.

2. Participation in the election of the president of India.

3. Approval of ordinances issued by the governor.

4. Enlargement of the jurisdiction of the state public service commission.

Select the correct answer using the code given below.

A. 1 and 2 only

B. 2 and 3 only

C. 1 and 4 only

D. 3 and 4 only

22. Bharatiya POSHAN KrishiKosh is a scheme initiated by the central

government. Consider the following statements:

1. It was launched by the Ministry of Women and Child Development along with World Bank.

2. It documents social, behavioural, and cultural practices which will help in promoting healthy dietary behaviours.

3. This project has only one component of the Development of a Food Atlas for India.

Which of the statements given above is/are correct?

A. 1 and 3 only

B. 2 only

C. 2 and 3 only

D. All of the above

23. Consider the following statements with reference to Cabinet Secretary:

1. He supervises the functions of ministries/departments.

2. He acts as a chief advisor to the Prime Minister on all aspects of administration and policy.

3. He is a Vice-Chairman of the Senior Selection Board which selects officers for the post of Joint Secretary in the Central Secretariat.

Which of the statements given above is/are correct?

Page 9: PRELIMS TEST SERIES - 2020 - Sleepy Classes

www.YouTube.com/SleepyClasses

www.sleepyclasses.com/

A. 1 and 3 only

B. 2 only

C. 2 and 3 only

D. 1 and 2 only

24. Which of the following agreements between the U.S and India has not been concluded yet?

1. Logistics Exchange Memorandum of

Agreement (LEMOA).

2. Communications Compatibility and Security Agreement (COMCASA).

3. Basic Exchange and Cooperation Agreement for Geo-spatial Cooperation (BECA)

Select the correct answer using the code given below.

A. 2 only

B. 3 only

C. 1 and 3 only

D. 2 and 3 only

25. Consider the following states mentioned below:

1. Gujarat

2. Himachal Pradesh

3. Madhya Pradesh

4. Maharashtra

5. Odisha

Which of the above States doesn’t have Scheduled Areas under the Fifth Schedule of the Constitution?

A. 1 and 2 only

B. 3, 4 and 5 only

C. 1, 3,4 and 5 only

D. None of the above

26. Under the Preventive detention Law in India, consider the following:

1. Constitution provides that the detenu should be afforded an opportunity to make a representation against the detention order at the earliest opportunity.

2. The detention of a person cannot exceed six months unless the arresting authority finds a sufficient cause for extended detention.

3. Only the centre can make laws with respect to Law & Order in states.

Which of the statements given above is/are correct?

A. 1 only

B. 1 and 2 only

C. 2 and 3 only

D. 1, 2 and 3

27. Consider the following statements with respect to the newly constituted system of Electoral Bonds in India.

1. These can be bought either by the companies or by any individuals.

2. These can be used to donate money to any political party in India.

3. These are issued by all the Public Sector Banks in India.

Which of the statements given above is/are correct?

A. 1 only

B. 2 and 3 only

C. 1 and 3 only

D. 1, 2 and 3

28. India has shown progress in which of the following parameters of Ease of Doing Business, Report of World Bank?

Page 10: PRELIMS TEST SERIES - 2020 - Sleepy Classes

www.YouTube.com/SleepyClasses

www.sleepyclasses.com/

1. Starting the business

2. Construction permits

3. Enforcing contracts

4. Resolving insolvency

Select the incorrect answer using the code given below:

A. 1 only

B. 3 only

C. 1 and 4 only

D. None of the above

29. The 44th Amendment Act, added which of the following Directive Principle of State Policy into Indian constitution?

1. Secure that the operation of the legal system promotes justice and shall provide free legal aid.

2. To secure opportunities for the healthy development of children.

3. Minimize the inequalities in income and endeavor to eliminate inequalities in status, facilities, and opportunities.

4. To protect and improve the environment and to safeguard forests and wildlife.

Select the correct answer using the code given below:

A. 1 and 3 only

B. 3 and 4 only

C. 3 only

D. 2 only

30. Consider the following statements regarding the Gram Nyayalayas:

1. It exercises the powers of Civil Courts

only.

2. It is not bound by the rules of evidence provided in the Indian Evidence Act, 1872.

3. Its presiding officer is appointed by the High Court in consultation with the State Government.

Which of the statements given above is/are correct?

A. 2 and 3 only

B. 2 only

C. 1 and 3 only

D. All of the above

31. Consider the following statements about

Rajya Sabha:

1. The seats are allotted to the states in the Rajya Sabha on the basis of the population.

2. Constitution has fixed the term of office of members of the Rajya Sabha for six years.

3. No UT’s are represented in Rajya Sabha.

Which of the statements given above is/are incorrect?

A. 1 and 2 only

B. 2 and 3 only

C. 3 only

D. None

32. Consider the following statements Which of the above statements with respect to the “National Broadband Mission (NBM)” are correct?

1. The mission is part of the National Digital Communications Policy, 2018.

2. It aims to provide broadband access to all villages by 2025.

Page 11: PRELIMS TEST SERIES - 2020 - Sleepy Classes

www.YouTube.com/SleepyClasses

www.sleepyclasses.com/

3. Its aim is to connect every Public and Private place of a village with an internet facility.

Choose the correct option

A. 1 only

B. 1 and 3 only

C. 1 and 2 only

D. 2 and 3 only

33. GOAL‘(Going Online as Leaders) is a digitally-enabled mentorship initiative of which one of the following?

A. Twitter

B. Facebook

C. Google

D. Microsoft

34. Consider the following statements

1. Provisions of the Citizenship Amendment Act will apply to the entire Indian subcontinent.

2. Only areas regulated through 'The Inner Limit Permit' will not be included.

3. The Citizenship (Amendment) Act grants citizenship to the Hindus, Christians, Sikhs, Buddhists, Jains, and Parsis who came from four countries Bordering India.

Which of the statements given above with reference to Citizenship (Amendment) Act, 2019 is/are not correct?

A. 1 and 2 only

B. 2 only

C. 2 and 3 only

D. All of the above

35. The committees which have recommended Unification of services for reforming Railways are:

1. Prakash Tandon Committee

2. Sam Pitroda Committee

3. Subramanian Swamy committee

Select the correct answer using the code given below:

A. 1 and 2 only

B. 2 and 3 only

C. 1 and 3 only

D. 1, 2 and 3

36. Consider the following statements regarding Union territories of India:

1. The Parliament can’t make laws on any subject including the State List for union territories.

2. Legislative assembly of UTs can‘t make laws on any subject of the Concurrent List.

3. Presently there are 9 UT’s in India.

Which of the statements given above is/are incorrect?

A. 1 and 2 only

B. 2 and 3 only

C. 1 and 3 only

D. All of the above

37. The National Air Quality Index mainly consists of:

1. Carbon dioxide

2. Carbon monoxide

3. Nitrogen dioxide

4. Sulfur dioxide

5. Methane

Page 12: PRELIMS TEST SERIES - 2020 - Sleepy Classes

www.YouTube.com/SleepyClasses

www.sleepyclasses.com/

Select the correct answer using the code given below:

A. 1, 2 and 3 only

B. 1, 4 and 5 only

C. 2, 3 and 4 only

D. 3, 4 and 5 only

38. Consider the following statements about Solid waste management techniques:

1. Pyrolysis is the process of burning waste in large furnaces at high temperatures.

2. Incineration is a process of combustion

in the absence of oxygen or the material burnt under controlled oxygen supply.

3. Composting is a biological process in which micro-organisms decompose organic waste in the presence of oxygen.

Which of the statements given above is/are incorrect?

A. 1 and 2 only

B. 2 and 3 only

C. 3 only

D. 1, 2 and 3 only

39. Consider the following statements:

1. It is one of the clean coal technologies were coal is converted into synthesis gas, also called Syngas.

2. Syngas is a mixture of hydrogen, Sulphur dioxide, and Ozone.

3. India’s first coal gasification based fertilizer plant is to be set up in Karnataka.

Which of the statements given above is/are correct about Syngas?

A. 1 only

B. 1 and 3 only

C. 2 and 3 only

D. 1, 2 and 3

40. Which of the following statements with respect to the Palm Oil industry are incorrect?

1. India is the world’s largest exporter of palm oil.

2. Indonesia and Malaysia contribute more than 85% of the world's Import of palm oil.

3. Kerala is the leading palm oil-producing state in India.

Select the correct code?

A. 1 and 2 only

B. 1 and 3 only

C. 2 and 3 only

D. 1, 2 and 3

41. South Asia’s first cross border Petroleum Products Pipeline project has been signed between India and:

A. Bhutan

B. Nepal

C. Bangladesh

D. Sri Lanka

42. Which of the following renewable sources of energy has the highest installed capacity in India?

A. Solar power

B. Wind power

C. Hydropower

D. Biopower

43. Consider the following the statements

Page 13: PRELIMS TEST SERIES - 2020 - Sleepy Classes

www.YouTube.com/SleepyClasses

www.sleepyclasses.com/

1. Climate forcings are factors in the climate system that either increase or decrease the effects to the climate system.

2. Negative forces such as excess greenhouse gases warm the earth.

3. Positive forces such as the effects of most aerosols and volcanic eruptions cool the earth.

Which of the above-mentioned statements is/are correct regarding Carbon Forcings?

A. 1 only

B. 1 and 2 only

C. 2 and 3 only

D. 1, 2 and 3

44. The term Geotail is being frequently used in news now a day’s, Consider the following statements about the same:

1. It is a region in space.

2. It exists as a result of the interactions between the Sun and the Moon.

3. Once every 29 days, the Moon traverses the geotail for about six days.

Which of the statements given above is/are incorrect?

A. 1 and 2 only

B. 2 only

C. 1 and 3 only

D. 1, 2 and 3

45. Consider the following statements about the Project Tiger in India:

1. Project Tiger is being implemented in more than half of Indian states.

2. As per the 2018 Tiger Census, Uttrakhand has the highest number of tiger populations in the country.

3. Out of all the Tiger reserves, the Bandhavgarh Tiger Reserve is the oldest one.

Which of the statements given above is/are correct?

A. 1 only

B. 2 and 3 only

C. 1 and 3 only

D. 3 only

46. Consider the following statements about KUSUM scheme:

1. It is an initiative of the Ministry of

Agriculture & Farmers' Welfare.

2. Under this scheme, farmers can avail of subsidies for the purchase of solar-powered pumps.

3. The main aim of this scheme is to install on-grid solar pumps only. Which of the above statements is/are correct with respect to KUSUM (Kisan Urja Suraksha evam Utthaan Mahabhiyan) scheme:

Select the correct option

A. 1 and 2 only

B. 2 only

C. 2 and 3 only

D. All of the above

47. Consider the following statements regarding the Emissions Gap Report 2019:

1. G20 nations collectively account for 78 per cent of all global greenhouse gas emissions.

2. The global GHG emission should fall by 7.6 per cent each year from 2020 to 2030 to meet the 1.5°C Paris climate agreement target.

3. This report is published by UNDP.

Page 14: PRELIMS TEST SERIES - 2020 - Sleepy Classes

www.YouTube.com/SleepyClasses

www.sleepyclasses.com/

Which of the statements given above is/are correct?

A. 1 and 2 only

B. 1 and 3 only

C. 2 and 3 only

D. All of the above

48. Consider the following statements:

1. Its objective is to improve the livelihoods of forest-dependent communities in the Central Indian Highlands.

2. It is financed by the centre and states in varying proportions.

3. It is implemented under the National Mission for Green India. Which of the statements given above is/are correct with reference to Ecosystem Service Improvement Project :

Choose the correct option

A. 1 and 2 only

B. 3 only

C. 1 and 3 only

D. 1, 2 and 3

49. Consider the following statements with

reference to Indian Rhino Vision 2020 (IRV 2020) programme:

1. It aims to attain a wild population of at least 3,000 greater one-horned rhino in Assam by the year 2020.

2. It is the lone initiative of Assam‘s Forest Department in collaboration with Government of India.

3. It involves creating new habitats for Rhino by spreading them over different

protected areas in all N-E States.

Which of the statements given above is/are correct?

A. 1 only

B. 2 and 3 only

C. 1 and 3 only

D. 1, 2 and 3

50. The term “Common but Differentiated Responsibilities” is related with:

A. Climate Change Mitigation Efforts

B. Solving NPA crisis

C. center-state relation and Good Governance

D. Eliminating Single Use Plastic

51. Bio Ethanol is being promoted now a day’s as an alternative source of fuel. Which of the following crops is/are the source for the same?

1. Sugarcane

2. Wheat

3. Sorghum

4. Tendu leaves

Select the correct answer using the code is given below:

A. 3 only

B. 1 and 2 only

C. 1 and 3 only

D. 1, 2 and 3

52. Fuel cells are being encouraged to be used as an alternative to traditional cell for

generating electricity. Consider the following statements for the same:

1. It is a device that converts chemical potential energy into electrical energy.

2. It produces electricity in the form of Alternating Current.

3. It produces Water, Heat, and just a small fraction of CO2 as a by-product.

Page 15: PRELIMS TEST SERIES - 2020 - Sleepy Classes

www.YouTube.com/SleepyClasses

www.sleepyclasses.com/

Which of the statements given above is/are incorrect?

A. 1 and 2 only

B. 2 and 3 only

C. 1 and 3 only

D. None of the above

53. Consider the following statements:

1. It is a global partnership including governments and Indigenous people only. Aiming to reduce emissions from forest degradation.

2. The Global Environment Facility (GEF) makes a minimum financial contribution of $5 million to FCPF every year.

3. Its objective is to assist countries in their REDD+ efforts by providing them with financial and technical assistance. Which of the statements given above is/are correct With reference to Forest Carbon Partnership Facility?

Select the correct option

A. 3 only

B. 1 and 3 only

C. 2 and 3 only

D. 1, 2 and 3

54. The bicuspid valve is located between of the following chambers of the heart?

A. Left Atrium and Left Ventricle

B. Right Atrium and Right Ventricle

C. Left Atrium and Right Atrium

D. Left Ventricle and Right Ventricle

55. The government recently launched the I-

STEM Portal. It is related with

A. Promoting judicious use of water

B. Intellectual property rights

C. Research and development facilities

D. Zero budget natural farming

56. Which of the following are categorised as dwarf planetsGoblin

1. Bennu

2. Ceres

3. Didymos

Select the correct option

A. 1 and 2

B. 3 and 4

C. 1 and 3

D. All of the above

57. Which of the following are correct in the context of X-rays?

1. Only gamma-rays carry more energy than the X-rays

2. X-rays do not penetrate the Earth’s atmosphere

Choose the correct option

A. 1 only

B. 2 only

C. Both 1 and 2

D. Neither 1 nor 2

58. Which of the following are humanoids that are deployed by various countries in space?

1. Fedor

2. Kirobo

3. Robonaut 2

Select the correct option

A. 1 and 3

B. 2 and 3

Page 16: PRELIMS TEST SERIES - 2020 - Sleepy Classes

www.YouTube.com/SleepyClasses

www.sleepyclasses.com/

C. 1 and 3

D. 1, 2 and 3

59. Consider the following statements in the context of plants outside earth

1. Cotton, rapeseed, and potato seeds have sprouted on the moon

2. Such attempts to grow plants in the International Space Station have not been successful yet.

Select the correct option

A. 1 only

B. 2 only

C. Both 1 and 2

D. Neither 1 nor 2

60. GPS Aided GEO Augmented Navigation (GAGAN) is used in which of the

following

1. Aviation

2. Deep-sea fishing

3. Railways

Choose the correct option

A. 1 and 2 only

B. 1 and 3 only

C. 1 only

D. All of the above

61. Goldschmidtite is

A. a new gold ore

B. the mineral from Earth’s mantle

C. a plant that takes in oxygen

D. a highly efficient nuclear fuel

62. Which of the following statement(s) is/are correct in the context of Lymphatic Filariasis

1. India has launched a triple-drug therapy regimen to eliminate lymphatic filariasis from the country by 2021

2. Short-term tourists are at comparatively lower risk than the local people

3. It is only spread by Culex mosquitoes

Select the correct option

A. 1 and 3 only

B. 2 and 3 only

C. 1 and 2 only

D. 1, 2 and 3

63. ‘Abhyas’ has been in news recently. It relates to

A. disaster management mock drill

B. preparing poor students for competitive exams

C. preparatory portal for unforeseen health emergencies

D. drone by DRDO

64. Which of the following fall under the category of anti-tank missiles

1. Nag

2. Spike

3. Strum Ataka

4. Javelin

Select the correct option

A. 1 and 2 only

B. 1, 2 and 3 only

C. 1, 2 and 4 only

D. 1, 2, 3 and 4

65. Bhashan Char Island, sometimes in news is situated at the mouth of river

Page 17: PRELIMS TEST SERIES - 2020 - Sleepy Classes

www.YouTube.com/SleepyClasses

www.sleepyclasses.com/

A. Meghna

B. Irrawaddy

C. Kaladan

D. Padma

66. Beijing Declaration and Platform for Action, often seen in the news, is (UPSC CSE 2015)

A. a strategy to tackle regional terrorism,

an outcome of a meeting of SCO

B. a plan of action for sustainable economic growth in the Asia-Pacific region, an outcome of the deliberations of the Asia-Pacific Forum

C. an agenda for women’s empowerment, an outcome of a World Conference convened by the United Nations

D. a strategy to combat wildlife trafficking, a declaration of the East Asia Summit

67. In the wake of the COVID-19 pandemic, which of the following has decided to issue Corona bonds?

A. IMF

B. World Bank

C. European Union

D. RCEP

68. Which one of the following is the best description of ‘INS Astradharini’, that was in the news recently? (UPSC CSE 2016)

A. Amphibious warfare ship

B. Nuclear-powered submarine

C. Torpedo launch and recovery vessel

D. Nuclear-powered aircraft carrier

69. What is IndARC?

A. Air missile defense system to be developed by India

B. Device to treat cancer

C. Neutrino observatory

D. Indigenous particle accelerator developed by India

70. Which of the following statements are correct with respect to Neolithic period?

1. Mud brick houses were built instead of grass huts during Neolithic period.

2. Evidence of pottery is absent from the Neolithic culture.

3. The people of Neolithic Age used clothes made of cotton and wool.

4. Domestication of animals started during Neolithic culture.

Select the correct code.

A. 1 and 3 only

B. 2 and 4 only

C. 2 and 3 only

D. 1 and 4 only

71. Consider the following statements with reference to the condition of women in Rig Vedic society.

1. Women were not allowed to wear garments made of cotton.

2. Child marriages were banned and the practice of sati was absent.

3. Women were not given equal opportunities as men for their spiritual and intellectual development.

Which of the above statements are correct?

A. 1 only

B. 1 and 3 only

C. 2 only

D. 3 only

Page 18: PRELIMS TEST SERIES - 2020 - Sleepy Classes

www.YouTube.com/SleepyClasses

www.sleepyclasses.com/

72. Senni, Valavan and Killi were famous titles assumed by

A. Pandya Kings

B. Temple priests

C. Chola Kings

D. Chera kings

73. Which of the following statements are correct with reference to the Fa Hien’s account of India?

1. His account was a portrayal of political affairs of Indian kingdoms.

2. He refers to the Gangetic valley as the ‘land of Brahmanism’.

3. He indicates the importance of the seven precious substances for Buddhist worship.

Select the correct answer using the codes below.

A. 1 and 3 only.

B. 2 only.

C. 3 only.

D. 2 and 3 only.

74. ‘Hiranya-garbha’ sacrifice was done during Rashtrakuta rule. It was done in order to

A. Declare war.

B. Convert the caste of an individual to Kshatriya.

C. Celebrate the birthday of king.

D. None of the above.

75. Consider the following statements with respect to Amir Khusrau.

1. He was a mystic and a spiritual disciple of Nizamuddin Auliya of Delhi.

2. His Khaliq-e-bari is about the achievements of Khilji rulers.

Which of the above statements are correct?

A. 1 only.

B. 2 only.

C. Both 1 and 2.

D. None of the above.

76. The fourth Buddhist Council was convened in Kashmir by Kanishka under the chairmanship of

A. Vasumitra

B. Moggaliputta Tissa

C. Aśvaghoṣ a

D. Mahakasapa

77. During 16th century, “Kunjali Marakkar”

was the title given to

A. Royal poets of southern kingdoms.

B. Naval chief of the Zamorin

C. Village heads who were also

landowners

D. Maratha’s army commander.

78. What were the important provisions in the Regulating act of 1773?

1. Provision was made in the Act for the establishment of a Supreme Court at Bombay.

2. A council of four members was appointed to assist the Governor-General.

3. The tenure of Governor-General of Fort William was extended from two year to four years.

Select the correct answer using the codes below.

A. 1 and 2 only.

Page 19: PRELIMS TEST SERIES - 2020 - Sleepy Classes

www.YouTube.com/SleepyClasses

www.sleepyclasses.com/

B. 3 only

C. 2 and 3 only.

D. 2 only

79. Consider the following statements with reference to Khilafat Movement.

1. The chief cause of the Khilafat Movement was the defeat of Turkey in the First World War.

2. Chittaranjan Das was associated with this movement.

Which of the above statements is/are correct?

A. 1 only

B. 2 only

C. Both 1 and 2

D. None of the above.

80. The governor-General who convinced the ruler of Kashmir, Ranjit Singh to sign the Indus Navigation Treaty was

A. Lord William Bentinck

B. Lord Amherst

C. Lord Hastings

D. Lord Auckland

81. Which of the following statements are correct with reference to the Partition of Bengal (1905)?

A. “Surat Split” was also the cause of partition.

B. New provinces of East Bengal and Assam were created after the partition of Bengal.

C. Both 1 and 2.

D. None of the above.

82. Consider the following statements with reference to the Factory Act of 1881.

1. It was introduced by Lord Ripon to improve the service condition of the factory workers in India.

2. It categorised organized and unorganized industries in India.

3. Following this act, a Factory Commission was appointed in 1885.

Which of the above statements is/are correct?

A. 1 and 2 only.

B. 2 only.

C. 1 and 3 only.

D. 1, 2 and 3.

83. Vedaranyam Salt Satyagraha March was undertaken by who among the following?

A. M.K. Gandhi

B. Periyar E.V.R.

C. Rajagopalachari

D. Sarojini Naidu

84. Consider the following statements

1. Dyarchy was introduced in the provinces. Provincial subjects were divided into “Reserved Subjects” and “Transferred Subjects”.

2. A bicameral (Two Chambers) legislature was set up at the centre. It consisted of the Council of States and the Legislative Assembly.

The above statements refers to

A. Montague-Chelmsford Reforms of 1919

B. Minto- Morley Reforms of 1909

C. Indian Councils Act of 1892

D. The Government of India Act of 1935

85. During whose tenure, following events occurred?

1. Poona Pact

Page 20: PRELIMS TEST SERIES - 2020 - Sleepy Classes

www.YouTube.com/SleepyClasses

www.sleepyclasses.com/

2. 2nd and 3rd Round Table Conference

3. Government of India Act 1935

Select the Correct Option

A. Lord Mayo

B. Lord Irwin

C. Lord Willingdon

D. Lord Linlithgow

86. The social base of the Swadeshi movement included which of the following groups?

1. Zamindari

2. Students

3. Women

4. Muslim peasantry

Choose the correct option:

A. 2 and 3 only

B. 2,3 and 4 only

C. 2 and 4 only

D. 1,2 and 3 only

87. Bosphorus Strait is between:

A. Caribbean Sea and North Atlantic

B. Gulf of Aden and the Red Sea

C. Gulf of Oman and the Persian Gulf

D. Black Sea to Sea of Marmara

88. The brahmaputra ,Irrawady,and Mekong rivers originate in Tibet and flow through

narrow and parallel mountain ranges in their upper reaches .Of these rivers , Brahmaputra makes a “U” turn in its course to flow into India. This “U” turn is due to:

A. Uplift of folded Himalayan series

B. Syntaxial bending of Geologically young Himalyas

C. Geo Tectonic Disturbance in the tertiary folded mountain chains

D. Both (a) and (b) above

89. Identify the drainage pattern is made by

the Ganga River system among the following:

A. Trellis Drainage Pattern

B. Angular Drainage Pattern

C. Rectangular Drainage Pattern

D. Radial Drainage Pattern

90. Recently, a Dutch government’s proposal to build two mega dams to separate the

North Sea from the Atlantic Sea, is being hailed as a future solution to defend northern Europe from rising sea levels. The project is referred to as “ NEED- Northern European Enclosure Dam”.

Which one among the following is marked incorrectly in context with the same?

1. France and England: NEED-North

2. Scotland and Norway: NEED-South

Select the Correct Option

A. Only 1

B. Only 2

C. Both 1 and 2

D. Neither 1 nor 2

91. “Room of the River Project” has been in news recently.

Find the incorrect statements in context with the same:

A. The origin of the project can be traced back to Japan.

B. It is to be replicated in Kerala’s Kuttanad as a flagship project

Page 21: PRELIMS TEST SERIES - 2020 - Sleepy Classes

www.YouTube.com/SleepyClasses

www.sleepyclasses.com/

C. It is a flood mitigation initiative which is aimed at protecting areas adjoining rivers from routine flooding and improving water management systems in delta regions.

D. None of the above.

92. “ Shannon-Weiner Index” has been in news recently.

Identify the incorrect statements in

context with the same?

1. It has recently been used in India State of the Forest Report 2019.

2. The index is used for measuring species richness and abundance among various habitats.

Select the Correct Option

A. Only 1

B. Only 2

C. Both 1 and 2

D. Neither 1 nor 2

93. As per Economic Survey 2020 ; find the

correct statements:

1. The agricultural credit flow target for 2019-20 has been fixed at Rs 13.5 lakh crore.

2. It is observed that credit is low in the Southern states whereas it is highest in North Eastern, Hilly and Eastern States

Select the Correct Option

A. Only 1

B. Only 2

C. Both 1 and 2

D. Neither 1 nor 2

94. “SDG India Index 2019” is frequently in news.

Find the correct statements in context with the same:

1. It is more comprehensive than SDG India Index 2018 and highlights the progress being made by the States/UTs on a wider set of 100 indicators spread across 16 goals

2. India’s score in the composite SDG India Index 2019 has improved from 57 in 2018 to 60 in 2019.

3. The goals that demand special attention are – 2 (Zero Hunger) and 5 (Gender Equality).

Select the Correct Option

A. 1 and 2 only

B. 2 and 3 only

C. 1 and 3 only

D. All of the above

95. Which of the following is matched incorrectly?

1. Chang’e-4: China

2. Hayabusa2: Japan

3. SnowEx : Russia

4. Voyager-2 : NASA

Select the Correct Option

A. Only 1

B. Only 2

C. Only 3

D. Only 4

96. Identify the tectonic Lakes among the following:

1. Wular Lake

2. Lake Baikal

3. Caspian Sea

Select the Correct Option

Page 22: PRELIMS TEST SERIES - 2020 - Sleepy Classes

www.YouTube.com/SleepyClasses

www.sleepyclasses.com/

A. 2 only

B. 3 only

C. 2 and 3 only

D. 1 ,2 and 3

97. Jet streams are a narrow variable band of very strong predominantly westerly air currents encircling the globe several miles above the earth. Which of the following are incorrect in context with the same?

1. Jet streams help in maintenance of latitudinal heat balance by mass exchange of air.

2. Jet streams are used by aviators if they have to fly in the direction of the flow of the jet streams, and avoid them when flying in opposite direction.

Select the Correct Option

A. Only 1

B. Only 2

C. Both 1 and 2

D. Neither 1 nor 2

98. Which of the following are incorrect with respect to Ecocline:

1. Thermocline -Based on difference in water temperature

2. Chemocline - Based on difference in density

3. Halocline - Based on difference in water salinity.

4. Pycnocline - Based on difference in chemical concentration

Select the Correct Option

A. 1 and 3 only

B. 2 and 4 only

C. 1,3 and 4

D. 2 only

99. Which of the following passes connect Chumbi Valley to Sikkim?

1. Nathu La

2. Jalep La

3. Shipki La

4. Bomdi La

Select the Correct Option

A. 1 Only

B. 1 and 2 only

C. 1, 2 and 3 only

D. 1, 2, 3 and 4

100. Identify the correct statements in context with “Seed Replacement Ratio”:

1. Seed Replacement Ratio is a measure of how much of the total cropped area was sown with certified seeds in comparison to farm saved seeds.

2. It represents the access of farmers to quality seed and inversely proportional to productivity of farming.

Select the Correct Option

A. Only 1

B. Only 2

C. Both 1 and 2

D. Neither 1 nor 2

Page 23: PRELIMS TEST SERIES - 2020 - Sleepy Classes

www.YouTube.com/SleepyClasses

www.sleepyclasses.com/

Full Length III

1. Why Net Domestic Product (NDP) is sometimes considered as a realistic assessment of an economy’s resources than a Gross Domestic Product (GDP).

Consider the following statements:

1. NDP takes into account taxes and subsidies paid which GDP does not.

2. NDP accounts for economic depreciation which GDP does not.

3. NDP counts the Net income earned by all the Indians living anywhere in the world.

Which of the above is/are correct?

A. 1 and 3 only

B. 2 only

C. 1 and 2 only

D. Neither 1 nor 2

Answer: B

Explanation:

NDP is GDP minus depreciation.

Depreciation is the total value of the wear and tear that happened in the assets while the goods and services were being produced in an economy.

2. In which of the following circumstances the GDP per capita of a country will necessarily increase?

1. Gross capital formation has increased

2. Export goods have increased

3. Index of industrial production has increased

4. Higher remittances have been received in the previous year

Select the correct answer using the codes below.

1. 1 and 2 only

2. 1, 2, and 3 only

3. 2, 3, and 4 only

4. None of the above

Answer: D

Explanation:

If capital is formed, there can be more production and thus growth. Per capita income may or may not increase. Overall income may increase.

Exports boost domestic production. IIP increase signals rise in the manufacturing and allied sectors. Higher remittances lead to higher domestic spending and thus higher demand for goods and services.

But none of it can definitely point to a higher per capita income because no statement talks of population being

constant or population being outpaced by the above factors.

3. ASPIRE is one of the main schemes of government of India, Consider the

following statements about the same:

1. The scheme is promoted by the Ministry of Skill Development.

2. The scheme aims at setting up

Livelihood Business Incubators (LBIs) only.

3. It will be implemented in both Rural and Urban areas.

Which of the statements given above is/are incorrect?

A. 1 and 2 only

B. 2 and 3 only

Page 24: PRELIMS TEST SERIES - 2020 - Sleepy Classes

www.YouTube.com/SleepyClasses

www.sleepyclasses.com/

C. All of the above

D. None of the above

Answer: C

Explanation: All are incorrect.

A Scheme for Promotion of Innovation, Rural Industry and Entrepreneurship (ASPIRE) is promoted by the Ministry of Micro, Small and Medium Enterprises.

As per the Budget 2019-20, The Scheme for Promotion of Innovation, Rural Industry, and Entrepreneurship‘ (ASPIRE) would be consolidated for setting up of 80 Livelihood Business Incubators (LBIs) as well as 20 Technology Business Incubators (TBIs) in 2019-20 to develop 75,000 skilled entrepreneurs in agro-rural industry sectors.

4. Consider the following statements:

1. Shanta Kumar headed the High-Level Committee on the restructuring of the Food Corporation of India (FCI).

2. The central government announces the Minimum Support Price (MSP) based on the recommendations of the Cabinet Committee on Economic Affairs.

3. MSP is decided before the harvesting season for crops.

Which of the statements given above is/are correct?

A. 1 and 3 only

B. 1 only

C. 2 and 3 only

D. None of the above

Answer: B

Explanation: Due to various problems related to primary functions of FCI i.e. procurement,

storage, and distribution; the government constituted a high powered committee on FCI restructuring headed by Shanta Kumar in 2014.

Before each Rabi/Kharif crop season, the central government announces the Minimum Support Prices (MSP), based on the recommendations of the Commission for Agricultural Costs and Prices (CACP), which takes into consideration the cost of various agricultural inputs and the reasonable margin for the farmers for their produce.

5. G20 is a very important institution in today’s world politics, Consider the following statements about the same:

1. It is an organization that consists of Head of the governments and central bank governors of member countries.

2. The formation of it was conceived after the Asian Financial Crisis in 1997- 1998.

3. The G20 has representation from 19 countries and the European Union only.

Which of the statements given above is/are incorrect?

A. 1 only

B. 2 and 3 only

C. 1 and 3 only

D. 1 and 2 only

Answer: C

Explanation:

G20 consists of Finance Ministers and Central Bank Governors of the group of countries. After the Asian Financial Crisis in 1997- 1998, it was conceived for the formation of G20.

Page 25: PRELIMS TEST SERIES - 2020 - Sleepy Classes

www.YouTube.com/SleepyClasses

www.sleepyclasses.com/

The G20 is an informal group of 19 countries and the European Union, with representatives of the International Monetary Fund and the World Bank.

6. Consider the following statements about WTO’s different Boxes:

1. Green box subsidies are subject to de minimis limits.

2. Blue box subsidies do not distort trade, or at most cause minimal distortion.

3. Amber box contains any support that would cause trade and production distortions and the support that requires farmers to limit production.

Which of the statements given above is/are correct?

A. 1 and 3 only

B. 2 and 3 only

C. 1 and 2 only

D. None of the above

Answer: D

Explanation: Green box subsidies must not distort trade, or at most cause minimal distortion. They have to be government-funded and must not involve price support. They also include environmental protection and regional development programmes. Green box subsidies are allowed without limits.

Domestic support measures considered to distort production and trade fall into the Amber Box. These supports are subject to limits. “De minimis” level of subsidiesare allowed which are generally 5% of agricultural production for developed countries and10% for developing countries.

Any support that would normally be in the amber box, is placed in the blue box if the support also requires farmers to limit production.

At present, there are no limits on spending on blue box subsidies.

7. Consider the following statements about National Food Security Act (NFSA):

1. The Act gives legal entitlement to around 75% of urban and 50% of the rural population of India to receive highly subsidized food grains.

2. The foodgrains are allocated at the rate of 5 kg per person per month for priority household’s category and 35 kg per family per month for Antyodaya Anna Yojana families.

3. Highly subsidized prices of Rs 1, 2, and 3 per kg for nutritious-cereals, wheat, and rice respectively are charged.

Which of the statements given above is/are correct with respect to the National Food Security Act (NFSA)?

A. 1 and 2 only

B. 2 and 3 only

C. 1 and 3 only

D. 1, 2 and 3

Answer: B

Explanation: The government of India enacted the National Food Security Act (NFSA) in July 2013 which gives legal entitlement to 67% of the population covering 75% in rural areas and 50% in urban areas to receive highly subsidized foodgrains.

Under the Act, foodgrain is allocated at the rate of 5 kg per person per month for priority households category and 35 kg per family per month for Antoyadaya Anna Yojana (AAY) families at highly subsidized prices of

Page 26: PRELIMS TEST SERIES - 2020 - Sleepy Classes

www.YouTube.com/SleepyClasses

www.sleepyclasses.com/

Rs 1, 2 and 3per kg for Nutri-cereals, wheat, and rice respectively.

Coverage under the Act is based on the population figures of Census, 2011. The Act is now being implemented in all 36 States/UTs and covers about 81.35 crore persons.

8. Consider the following statements about Gender Budgeting?

1. It uses a budget as an entry point to apply a gender lens to the entire policy process.

2. It provides for a separate budget for women.

3. The Ministry of Finance is the nodal agency for its implementation.

Select the incorrect answer using the code given below:

A. 1 only

B. 1 and 2 only

C. 2 and 3 only

D. 1 and 3 only

Answer: C

Explanation: Gender budgeting is a tool for gender mainstreaming. It uses a budget as an entry point to apply a gender lens to the entire policy process.

A gender budget is not a separate budget for women. Instead, the gender budgets are an attempt to assess the government’s priorities as they are reflected through a budget and examine how they impact men and women.

The Ministry for Women and Child Development acts as the nodal ministry for ensuring gender budgeting. It is concerned with gender-sensitive formulations of legislations, allocation of resources, monitoring and impact

assessment of programmes and schemes.

9. With reference to the Equity Grant Scheme, consider the following statements:

1. Under this scheme, the government grants equity to individual investors Farmer producer organizations

2. This scheme is implemented by NABARD.

Which of the above statements is/are correct?

A. 1 only

B. 2 only

C. Both 1 and 2

D. Neither 1 nor 2

Answer: A

Explanation:

Under this scheme any farmer (Small/Marginal), the landless labourer can become a shareholder by contributing to Equity of the Farmer Producer Companies (FPC). The Equity Grant Fund has been set up with the primary objectives of:

Enhancing viability and sustainability of FPCs.

Increasing creditworthiness of FPCs.

Enhancing the shareholding of members to increase their ownership and participation in their FPC.It is being operated by Small Farmers Agri-Business Consortium (SFAC).

10. Consider the following statements about Money Market instruments:

1. The money market is a market for short term funds whose period of maturity is up to one year.

Page 27: PRELIMS TEST SERIES - 2020 - Sleepy Classes

www.YouTube.com/SleepyClasses

www.sleepyclasses.com/

2. Treasury bills are the short term equity instruments issued by the Government of India.

3. Commercial Paper (CP) is a secured money market instrument issued in the form of a promissory note.

Which of the statements given above is/are correct?

A. 1 and 2 only

B. 1 and 3 only

C. 1 only

D. 1, 2 and 3

Answer: C

Explanation: The money market is defined as

dealing in debt of less than one year.

It is a means for governments and corporations to keep their cash flow steady, and for investors to make a modest profit.

The capital market is dedicated to the sale and purchase of long-term debt and equity instruments.

Treasury bills or T-bills are money market instruments. They are short term debt instruments issued by the Government of India.

Commercial Paper (CP) is an unsecured money market instrument issued in the form of a promissory note.

It was introduced in India in 1990 with a view to enabling highly rated corporate borrowers to diversify their sources of short-term borrowings and to provide an additional instrument to investors. Subsequently, primary dealers and all-India financial institutions were also permitted to issue CP to enable them to meet their short term funding requirements for their operations

11. Consider the following statements with respect to the Infrastructure Investment Funds (InvITs):

1. InvITs pool small sums of money from many investors to invest in assets that give long term returns.

2. InvITs are suitable for high net worth individuals.

3. These funds are Alternate investment funds that are not registered with SEBI.

Which of the statements given above is/are correct?

A. 2 and 3 only

B. 3 only

C. 1 and 2 only

D. 1, 2 and 3 only

Answer: C

Explanation: An Infrastructure Investment Trust (InvITs) is like a mutual fund, which enables direct investment of small amounts of money from possible individual/ institutional investors in infrastructure to earn a small portion of the income as a return. InvITs work like mutual funds or real estate investment trusts (REITs).

Since the minimum investment amount in an IPO is Rs 10 lakh, InvITs are suitable for high net worth individuals, institutional and non-institutional investors like pension funds, foreign portfolio investors, mutual funds, banks, and insurance firms. InvITs are listed on exchanges just like stocks through IPOs

SEBI said the issuer will have to disclose objects of the issue, related-party transactions, valuation, financial details, review of credit rating, and grievance redressal mechanism in the placement document. The SEBI had first notified REITs and InvIT

Page 28: PRELIMS TEST SERIES - 2020 - Sleepy Classes

www.YouTube.com/SleepyClasses

www.sleepyclasses.com/

Regulations in 2014, allowing setting up and listing of such trusts which are popular in some advanced markets

12. Utkarsh 2022 scheme of Government of India is related to:

A. Roadmap to make India a 5 trillion economy

B. Merging of Public sector banks

C. Checking the liquidity problem of NBFC’s

D. Improve the regulation and supervision of the central bank

Answer: D

Explanation: Utkarsh 2022 is a medium-term strategy in line with the global central Banks plan to strengthen the regulatory and supervisory mechanism of the Central Bank.

13. Consider the following statements about Non-Banking Financial Companies (NBFCs):

1. NBFCs do not form part of the payment and settlement system and cannot issue cheques drawn on itself

2. NBFCs whose asset size is of Rs. 1000 cr or more as per the last audited balance sheet are considered as systemically important NBFCs.

3. NBFCs can accept time and demand deposits.

Which of the statements given above is/are correct?

A. 1 only

B. 1 and 3 only

C. 2 and 3 only

D. 1, 2 and 3

Answer: A

Explanation:

A NonBanking Financial Company (NBFC) is a company registered under

the Companies Act, 1956 engaged in the business of loans and advances,

acquisition of shares/stocks/bonds/debentures/securities issued by Government or local authority or other marketable securities of a like nature, leasing, hire-purchase, insurance business, chit business but does not include any institution whose principal business is that of agriculture activity, industrial activity, purchase or sale of any goods (other than securities), or providing any services and sale/purchase/construction of immovable property. NBFCs lend and make investments and hence their activities are akin to that of banks;

however, there are a few differences as given below:

NBFC cannot accept demand deposits;

NBFCs do not form part of the payment and settlement system and cannot issue cheques drawn on itself;

Deposit insurance facility of Deposit Insurance and Credit Guarantee Corporation is not available to depositors of NBFCs, unlike in the case of banks.

Systemically important NBFCs

are NBFCs whose asset size is of Rs.500 cr or more as per last audited balance sheet are considered as systemically important NBFCs.

The rationale for such classification is that the activities of such NBFCs will have a bearing on the financial stability of the overall economy.

14. Consider the following statements

Page 29: PRELIMS TEST SERIES - 2020 - Sleepy Classes

www.YouTube.com/SleepyClasses

www.sleepyclasses.com/

1. It mainly involves permanent work contracts.

2. It provides cheaper and more efficient services to the users.

Which of the above are incorrect about the Gig economy?

A. 1 only

B. 2 only

C. Both 1 and 2

D. Neither 1 nor 2

Answer: A

Explanation: The gig economy is based on flexible, temporary, or freelance jobs, often involving connecting with clients or customers through an online platform.

In a gig economy, large numbers of people work part-time or temporary positions. The result of a gig economy is cheaper, more efficient services, such as Uber or Airbnb, for those willing to use them.

Those who don't engage in using technological services such as the Internet tend to be left behind by the benefits of the gig economy.

In a gig economy, businesses save resources in terms of benefits, office space, and training.

15. Plastic parks are being set up at various locations in India. Consider the following statements about them:

1. It is a zone where used plastic materials are dumped.

2. Odisha has become the first state to start working on developing a Plastic Park.

Which of the statements given above is/are correct?

A. 1 only

B. 2 only

C. 1 and 2 only

D. Neither 1 nor 2

Answer: B

Explanation: A plastic park is an industrial zone devoted to plastic enterprises. It includes a whole range of companies required by the plastics processing community from material and machinery suppliers, plastic processing

companies, plastic recycling companies, etc.

The objective of setting up the plastic parks is to increase the competitiveness, polymer absorption capacity, and value addition in

the domestic downstream plastic processing industry.

They also serve the twin objective of investment attraction and employment generation.

The Central Government has decided to set up a Plastic Park in Jagatsinghpur district of

Odisha during 2019-20.

Three more Plastic Parks would be set up in Tinsukia (Assam), Raisen (M.P.) and Thiruvallur (Tamil Nadu).

16. Pigovian tax is best described by the term:

A. It is imposed on the profits of pig rearing enterprises.

B. It is imposed on the transactions having negative externalities.

C. It is a tax on all foreign exchange transactions.

D. It is a safeguarding tax against the foreign items being dumped in the country.

Answer: B

Explanation:

Page 30: PRELIMS TEST SERIES - 2020 - Sleepy Classes

www.YouTube.com/SleepyClasses

www.sleepyclasses.com/

A Pigovian tax is a fee that is assessed against private individuals or businesses for engaging in activities that create adverse side effects.

The Pigovian tax is meant to discourage activities that impose a net cost of production onto third parties and society as a whole.

According to Pigovian, negative externalities prevent a market economy from reaching equilibrium when producers do not internalize all costs of production.

This adverse effect might be corrected, he contended, by levying taxes equal to the externalized costs.

17. Consider the following statements :

1. The IMF’s primary purpose is to ensure the stability of the international monetary system.

2. IMF members cannot voluntarily exchange Special Drawing Rights (SDRs) for currencies among themselves.

3. Though India is a member of the IMF, it is not a founding member of it.

4. India has not taken any financial assistance from the IMF since 1993.

Which of the above statements regarding the International Monetary Fund (IMF) are correct?

A. 1, 2 and 3 only

B. 1, 3 and 4 only

C. 1 and 4 only

D. 1 and 3 only

Answer: C

Explanation: The IMF’s primary mission is to ensure the stability of the international monetary system—the system of exchange

rates and international payments that enables countries and their citizens to transact with each other.

The IMF issues an international reserve asset known as Special Drawing Rights or SDRs, that can supplement the official reserves of member countries. IMF members can voluntarily exchange SDRs for currencies among themselves.

India is a founder member of the IMF.

India has not taken any financial assistance from the IMF since 1993.

Repayments of all the loans taken from International Monetary Fund have been completed on 31 May 2000.

18. Consider the following statements with reference to the Debt to GDP ratio.

1. Debt to GDP ratio of states has increased in the recent Financial Years.

2. The N.K. Singh Committee report had recommended the ratio to be 40% for the States and 20% for the Centre, respectively, by 2023.

Which of the statements given above is/are correct?

A. 1 only

B. 2 only

C. Both 1 and 2

D. Neither 1 nor 2

Answer: A

Explanation: Outstanding liabilities of states, as a percent of GDP, has increased after 2014-15. The issuance of UDAY bonds in 2015-16 and 2016-17, farm loan waivers, and the implementation of pay commission awards have led to higher debt to GDP ratio.

The N.K. Singh-headed FRBM (Fiscal Responsibility and Budget Management) Review Committee

Page 31: PRELIMS TEST SERIES - 2020 - Sleepy Classes

www.YouTube.com/SleepyClasses

www.sleepyclasses.com/

report had recommended the ratio to be 40% for the Centre and 20% for the States, respectively, by 2023.

19. The Concurrent List of the 7th schedule of the constitution contains:

1. Forest

2. Wild animals and birds

3. Administration of justice

4. Education

5. Public order

Select the correct answer using the code given below:

A. 1, 3 and 5 only

B. 1, 2 and 3 only

C. 1, 2,3 and 4 only

D. 1, 2, 3, 4 and 4 only

Answer: C

Explanation:

42nd Constitutional Amendment Act (1976) restructured the Seventh Schedule ensuring that State List subjects like education, forest, protection of wild animals and birds, administration of justice, and weights and measurements were transferred to the Concurrent List. Public order is under the state list of the 7th schedule.

20. Consider the following statements:

1. The Commission for SCs and STs were bifurcated into two separate bodies by the 89th Constitutional Amendment Act.

2. Chairperson and members of the National Commission for SCs hold office for a term of five years.

3. They are appointed by a committee consists of P.M, leader of opposition in L.S and Speaker of L.S

Which of the statements given above is/are incorrect?

A. 1 and 2 only

B. 1 only

C. 2 and 3 only

D. None of the above

Answer: C

Explanation:

The 89th Constitutional Amendment Act of 2003 bifurcated the combined National Commission for SCs and STs into two separate bodies, namely, National Commission for Scheduled Castes (under Article 338) and National Commission for Scheduled Tribes

(under Article 338-(A).

The separate National Commission for SCs came into existence in 2004. It consists of a chairperson, a vice-chairperson, and three other members. They are appointed by the President by warrant under his hand and seal.

Their conditions of service and tenure of office are also determined by the President. Under the Rules, Chairperson and members hold office for a term of three years.

21. In which of the following matters the

power of Legislative Council of a state is not equal to Legislative Assembly :

1. Voting on the demands for grants.

2. Participation in the election of the president of India.

3. Approval of ordinances issued by the governor.

Page 32: PRELIMS TEST SERIES - 2020 - Sleepy Classes

www.YouTube.com/SleepyClasses

www.sleepyclasses.com/

4. Enlargement of the jurisdiction of the state public service commission.

Select the correct answer using the code given below.

A. 1 and 2 only

B. 2 and 3 only

C. 1 and 4 only

D. 3 and 4 only

Answer: A

Explanation: POSITION OF LEGISLATIVE COUNCIL

The constitutional position of the council (as compared with the assembly) can be studied from two

angles:

Equal with Assembly

Introduction and passage of ordinary bills. However, in case of disagreement between the two Houses, the will of the assembly prevails over that of the council.

Approval of ordinances issued by the governor.

Selection of ministers including the chief minister.

Enlargement of the jurisdiction of the state public service commission.

Unequal with Assembly

A Money Bill can be introduced only in the assembly and not in the council.

The council can only discuss the budget but cannot vote on the demands for grants (which is the exclusive privilege of the assembly).

The council does not participate in the election of the president of India and representatives of the state in the Rajya Sabha.

22. Bharatiya POSHAN KrishiKosh is a scheme initiated by the central government. Consider the following statements:

1. It was launched by the Ministry of Women and Child Development along with World Bank.

2. It documents social, behavioural, and cultural practices which will help in promoting healthy dietary behaviours.

3. This project has only one component of the Development of a Food Atlas for India.

Which of the statements given above is/are correct?

A. 1 and 3 only

B. 2 only

C. 2 and 3 only

D. All of the above

Answer: B

Explanation:

Ministry of Women and Child

Development along with Bill & Melinda Gates Foundation (BMGF) announced the Bhartiya Poshan Krishi Kosh (BPKK) in New Delhi on 18th November 2019. Bhartiya Poshan Krishi Kosh‖ project has two components – Development of a Food Atlas and Documentation of promising practices for Jan-Andolan for POSHAN Abhiyaan.

The project also documents social,

behavioural and cultural practices that promote and reinforce healthy dietary behaviours.

The project includes diverse data sources like National Sample Survey, Agri-Census, Soil Health Cards, ISRO‘s Advanced Wide Field Sensor and National Aeronautics and Space Administration‘s (NASA‘s) Moderate

Page 33: PRELIMS TEST SERIES - 2020 - Sleepy Classes

www.YouTube.com/SleepyClasses

www.sleepyclasses.com/

Resolution Imaging Spectro-Radio meter

23. Consider the following statements with reference to Cabinet Secretary:

1. He supervises the functions of ministries/departments.

2. He acts as a chief advisor to the Prime Minister on all aspects of administration and policy.

3. He is a Vice-Chairman of the Senior Selection Board which selects officers for the post of Joint Secretary in the Central Secretariat.

Which of the statements given above is/are correct?

A. 1 and 3 only

B. 2 only

C. 2 and 3 only

D. 1 and 2 only

Answer: B

Explanation:

The office of Cabinet Secretary was created in India in 1950. Its first occupant was N.R.Pillai. A Cabinet Secretary is the head of the Cabinet Secretariat.

He succeeded the Secretary to the Governor-General‘s Executive Council. He is given a top place among the civil servants in the official warrant of precedence. Thus, he is the senior-most civil servant in India.

The following points highlight the role, powers, and functions of a Cabinet Secretary.

He is the chief coordinator of Central administration. But, he has no supervisory function over ministries/departments.

He is a Chairman of the Senior Selection Board which selects officers for the post of Joint Secretary in the Central Secretariat.

He himself selects the officers for the posts of Secretary and Additional Secretary in the Central Secretariat.

He is the Chairman of the Committee of Secretaries on Administration which is set up to resolve inter-ministerial disputes.

He presides over the Conference of Chief Secretaries which is held annually.

He acts as a chief advisor to the Prime Minister on all aspects of administration and policy.

His sanction should be obtained by a minister before launching prosecution against the publisher or editor of a newspaper in cases of defamation.

He serves on occasions as a factotum when the Prime Minister deems necessary and calls him to be so.

He acts as an advisor and conscience keeper to all the civil servants. He advises and guides them on cases of inter-departmental difficulties.

He acts as a link between the Prime Minister‘s Office (PMO) and various administrative agencies and also between the civil service and the political system.

24. Which of the following agreements between the U.S and India has not been concluded yet?

1. Logistics Exchange Memorandum of Agreement (LEMOA).

2. Communications Compatibility and Security Agreement (COMCASA).

Page 34: PRELIMS TEST SERIES - 2020 - Sleepy Classes

www.YouTube.com/SleepyClasses

www.sleepyclasses.com/

3. Basic Exchange and Cooperation Agreement for Geo-spatial Cooperation (BECA)

Select the correct answer using the code given below.

A. 2 only

B. 3 only

C. 1 and 3 only

D. 2 and 3 only

Answer: B

Explanation:

In 2016, the US government declared India a major defence partner‖, which made India eligible for receiving defence technologies at par with those provided to the US‘ closest allies — the NATO+5.

During that same year, India signed the Logistics Exchange Memorandum

of Agreement (LEMOA), the first of the three foundational defence pacts that need to be signed by a country to obtain high-tech military hardware

from the US.

Last year, the Communications Compatibility and Security

Agreement (COMCAS(A) was also signed, and the remaining pact, the Basic Exchange and Cooperation Agreement for Geo-spatial Cooperation (BECA) will also be

signed soon.

25. Consider the following states mentioned below:

1. Gujarat

2. Himachal Pradesh

3. Madhya Pradesh

4. Maharashtra

5. Odisha

Which of the above States doesn’t have Scheduled Areas under the Fifth Schedule of the Constitution?

A. 1 and 2 only

B. 3, 4 and 5 only

C. 1, 3,4 and 5 only

D. None of the above

Answer: D

Explanation:

As per the provision under Article 244 (1) of the Constitution of India, the ‘Scheduled Areas’ are defined as ‘such areas as the President may by order declare to be Scheduled Areas’ – as per paragraph 6(1) of the Fifth Schedule of the Constitution of India.

At present, Scheduled Areas have been declared in the States of Andhra Pradesh, Telangana, Chhattisgarh, Gujarat, Himachal Pradesh, Jharkhand, Madhya Pradesh, Maharashtra, Odisha, and Rajasthan

26. Under the Preventive detention Law in India, consider the following:

1. Constitution provides that the detenu should be afforded an opportunity to make a representation against the detention order at the earliest opportunity.

2. The detention of a person cannot exceed six months unless the arresting authority finds a sufficient cause for extended detention.

3. Only the centre can make laws with respect to Law & Order in states.

Which of the statements given above is/are correct?

A. 1 only

B. 1 and 2 only

Page 35: PRELIMS TEST SERIES - 2020 - Sleepy Classes

www.YouTube.com/SleepyClasses

www.sleepyclasses.com/

C. 2 and 3 only

D. 1, 2 and 3

Answer: A

Explanation: Article 22 grants some protection

to persons who are arrested or detained under a preventive detention law. It provides that detenu should be afforded an opportunity to make a representation against the detention order at the earliest opportunity.

Article 22 also provides that detention of a person cannot exceed three months (not six) unless an advisory board reports sufficient cause for extended detention. The board is to consist of judges of a high court.

The Constitution has divided the legislative power with regard to preventive detention between the Parliament and the state legislatures.

The Parliament has exclusive authority to make a law of preventive detention for reasons connected with defence, foreign affairs, and the security of India.

Both the Parliament as well as the state legislatures can concurrently make a law of preventive detention for reasons connected with the security of a state, the maintenance of public order and the maintenance of supplies and services essential to the community.

27. Consider the following statements with respect to the newly constituted system of Electoral Bonds in India.

1. These can be bought either by the companies or by any individuals.

2. These can be used to donate money to any political party in India.

3. These are issued by all the Public Sector Banks in India.

Which of the statements given above is/are correct?

A. 1 only

B. 2 and 3 only

C. 1 and 3 only

D. 1, 2 and 3

Answer: A

Explanation:

As per provisions of the Scheme, electoral bonds may be purchased by a citizen of India or entities incorporated or established in India. A person being an individual can buy electoral bonds,

either singly or jointly with other individuals.

Only the registered political parties which have secured not less than one percent of the votes polled in the last Lok Sabha elections or the State Legislative Assembly are eligible to receive the Electoral Bonds.

These bonds have been available at specified branches of State Bank of India. They can be bought by the donor with a KYC-compliant account.

28. India has shown progress in which of the

following parameters of Ease of Doing Business, Report of World Bank?

1. Starting the business

2. Construction permits

3. Enforcing contracts

4. Resolving insolvency

Select the incorrect answer using the code given below:

A. 1 only

B. 3 only

C. 1 and 4 only

Page 36: PRELIMS TEST SERIES - 2020 - Sleepy Classes

www.YouTube.com/SleepyClasses

www.sleepyclasses.com/

D. None of the above

Answer: B

Explanation:

The World Bank recently released

its Ease of Doing Business Report, 2020. According to the report, New Zealand retained its 1st position whereas Somalia was ranked at 190th spot. India was placed at 63rd position this time (2019) out of 190 countries marking an improvement of 14 places from its 77th in 2018.

India’s score improved from 67.23 (2019) to 71.0 (2020). India for the third consecutive year was present in the list of 10 economies where the business climate has improved the most. The essential features of India’s performance this year are: India’s ranking improved basically on four parameters:

Starting a Business

Dealing with Construction Permits

Trading across Borders

Resolving Insolvency

29. The 44th Amendment Act, added Which of the following Directive Principle of

State Policy into Indian constitution?

1. Secure that the operation of the legal system promotes justice and shall provide free legal aid.

2. To secure opportunities for the healthy development of children.

3. Minimize the inequalities in income and endeavor to eliminate inequalities in status, facilities, and opportunities.

4. To protect and improve the environment and to safeguard forests and wildlife.

Select the correct answer using the code given below:

A. 1 and 3 only

B. 3 and 4 only

C. 3 only

D. 2 only

Answer: C

Explanation: 42nd Amendment Act of 1976 added that the state shall secure that the operation of the legal system promotes justice and shall provide free legal aid.

42nd Amendment Act of 1976 added that the state shall direct its policy towards securing that children are given opportunities and facilities to develop in a healthy manner.

44th Amendment Act of 1978 added that the State shall strive to minimize the inequalities in income and endeavour to eliminate inequalities in status, facilities and opportunities.

42nd Amendment Act of 1976 added that the state shall endeavor to protect and improve the environment and to safeguard forests and wildlife.

30. Consider the following statements

regarding the Gram Nyayalayas:

1. It exercises the powers of Civil Courts only.

2. It is not bound by the rules of evidence provided in the Indian Evidence Act, 1872.

3. Its presiding officer is appointed by the High Court in consultation with the State Government.

Which of the statements given above is/are correct?

A. 2 and 3 only

Page 37: PRELIMS TEST SERIES - 2020 - Sleepy Classes

www.YouTube.com/SleepyClasses

www.sleepyclasses.com/

B. 2 only

C. 1 and 3 only

D. All of the above

Answer: B

Explanation: The salient features of the Gram Nyayalayas Act- 2008 are as follows-

The Gram Nyayalaya is a court of Judicial Magistrate of the first class and its presiding officer (Nyayadhikari) is appointed by the State Government in consultation with the High Court.

The Gram Nyayalayas is established for every Panchayat at an intermediate level or a group of contiguous

Panchayats at intermediate level in a district or where there is no Panchayat at an intermediate level in any State, for a group of contiguous Panchayats. The Gram Nyayalaya is a mobile court and exercises the powers of both Criminal and Civil Courts.

The seat of the Gram Nyayalaya will be located at the headquarters of the intermediate Panchayat, they will go to villages, work there and dispose of the cases.

The Gram Nyayalaya shall try criminal cases, civil suits, claims, or disputes which are specified in the First Schedule and the Second Schedule to the Act.

The Gram Nyayalaya are not bound by the rules of evidence provided in the Indian Evidence Act, 1872 but are guided by the principles of natural justice and subject to any rule made by the High Court.

31. Consider the following statements about Rajya Sabha:

1. The seats are allotted to the states in the Rajya Sabha on the basis of the population.

2. Constitution has fixed the term of office of members of the Rajya Sabha for six years.

3. No UT’s are represented in Rajya Sabha.

Which of the statements given above is/are incorrect?

A. 1 and 2 only

B. 2 and 3 only

C. 3 only

D. None

Answer: B

Explanation: The representatives of states in the Rajya Sabha are elected by the elected members of state legislative assemblies. The election is held in accordance with the system of proportional representation by means of the single transferable vote. The seats are allotted to the states in the Rajya Sabha on the basis of the population.

The Rajya Sabha (first constituted in 1952) is a continuing chamber, that is, it is a permanent body and not subject to dissolution. However, one-third of its members retire every second year.

Their seats are filled up by fresh elections and presidential nominations at the beginning of every third year. The retiring members are eligible for re-election and renomination any number of times.

The Constitution has not fixed the term of office of members of the Rajya Sabha and left it to the Parliament.

Accordingly, the Parliament in the Representation of the People Act (1951) provided that the term of office of a member of the Rajya Sabha shall be six years. The act also empowered

Page 38: PRELIMS TEST SERIES - 2020 - Sleepy Classes

www.YouTube.com/SleepyClasses

www.sleepyclasses.com/

the president of India to curtail the term of members chosen in the first Rajya Sabha.

32. Consider the following statements Which of the above statements with respect to the “National Broadband Mission (NBM)” are correct?

1. The mission is part of the National Digital Communications Policy, 2018.

2. It aims to provide broadband access to all villages by 2025.

3. Its aim is to connect every Public and Private place of a village with an internet facility.

Choose the correct option

A. 1 only

B. 1 and 3 only

C. 1 and 2 only

D. 2 and 3 only

Answer: A

Explanation:

The central government recently launched the National Broadband Mission, which envisages an investment of Rs. 7 lakh crore in the next 4 years from both government and industry to connect the country digitally at a faster pace. The mission, launched by Union Telecom Minister, is part of the National Digital

Communications Policy, 2018.

The mission envisages Rs. 7 lakh crore investments from government and private sector together over the next 3 to 4 years.

Some of the objectives of the National Broadband Mission which is structured with a strong emphasis on the three principles of universality,

affordability, and quality are:

Broadband access to all villages by 2022

Facilitate universal and equitable access to broadband services for across the country and especially in rural and remote areas

Laying of incremental 30 lakhs route km of Optical Fiber Cable and an increase in tower density from 0.42 to 1.0 tower per thousand of the population by 2024

Significantly improve the quality of services for mobile and internet

Develop innovative implementation models for Right of Way (RoW) and to work with States/UTs for having consistent policies pertaining to expansion of digital infrastructure including for RoW approvals required for laying of OFC

Develop a Broadband Readiness Index (BRI) to measure the availability of digital communications infrastructure and a conducive policy ecosystem within a State/UT.

Investment from stakeholders of USD 100 billion (Rs 7 Lakh Crore) including Rs 70,000 crore from Universal Service Obligation Fund (USOF).

33. GOAL‘(Going Online as Leaders) is a digitally-enabled mentorship initiative of which one of the following?

A. Twitter

B. Facebook

C. Google

D. Microsoft

Answer: B

Explanation:

Page 39: PRELIMS TEST SERIES - 2020 - Sleepy Classes

www.YouTube.com/SleepyClasses

www.sleepyclasses.com/

Facebook announced the second phase of its GOAL‘ initiative under which the social networking giant is helping tribal girls from across India to become village-level digital young leaders for their communities.

The second phase of the programme — launched by Tribal Affairs Minister Arjun Munda — will digitally mentor 5,000 young women in India‘s tribal-dominated districts. Going Online as Leaders (GOAL) connects underprivileged young women from tribal areas with senior expert mentors in the areas of business, fashion and arts to learn digital and life skills.

The initiative will include weekly one-to-one mentoring sessions that focus on a range of skills such as digital literacy, entrepreneurship and online safety. Overall, more than 2,00,000 hours of guidance will be provided using the Facebook family of apps including WhatsApp and Messenger.

Following the programme, the participants will graduate to the GOAL alumni and will continue to

receive support and guidance from the Ministry of Tribal Affairs and Facebook.

34. Consider the following statements

1. Provisions of the Citizenship Amendment Act will apply to the entire Indian subcontinent.

2. Only areas regulated through 'The Inner Limit Permit' will not be included.

3. The Citizenship (Amendment) Act grants citizenship to the Hindus, Christians, Sikhs, Buddhists, Jains, and Parsis who came from four countries Bordering India.

Which of the statements given above with reference to Citizenship (Amendment) Act, 2019 is/are not correct?

A. 1 and 2 only

B. 2 only

C. 2 and 3 only

D. All of the above

Answer: D

Explanation: Citizenship (Amendment) Act makes it easier for Hindus, Sikhs, Buddhists, Jains, Parsis, and Christians to face religious persecution in Pakistan, Bangladesh, and Afghanistan, to seek Indian citizenship. The Citizenship (Amendment) Act grants citizenship to the Hindus, Christians, Sikhs, Buddhist, Jains, and Parsis — from Afghanistan, Pakistan and Bangladesh who had arrived in India before 31 December 2014.

Provisions of the amendments to the citizenship Act would not be applicable to the tribal areas of Assam, Meghalaya, Mizoram, and Tripura, as included in the Sixth Schedule to the Constitution.

The areas regulated under 'The Inner Line' permit have been exempted, Manipur also brought under the Inner Limit Permit (ILP) regime.

35. The committees which have recommended Unification of services for reforming Railways are:

1. Prakash Tandon Committee

2. Sam Pitroda Committee

3. Subramanian Swamy committee

Select the correct answer using the code given below:

A. 1 and 2 only

Page 40: PRELIMS TEST SERIES - 2020 - Sleepy Classes

www.YouTube.com/SleepyClasses

www.sleepyclasses.com/

B. 2 and 3 only

C. 1 and 3 only

D. 1, 2 and 3

Answer: A

Explanation:

The Union Cabinet chaired by Prime Minister has recently approved a transformational organizational restructuring of the Indian Railways. This historic reform will help in achieving the Government‘s vision of making Indian Railways the growth engine of India's Vikas Yatra. The reforms include:

Unification of the existing eight Group A services of the Railways into a Central Service called Indian Railway Management

Service (/RMS)

Re-organization of Railway Board on functional lines headed by CRB with four Members and some Independent Members

The existing service of Indian Railway Medical Service

(IRMS) to be consequently renamed as Indian Railway Health Service (IRHS) Unification of services has been recommended by various committees for reforming Railways including - PrakashTandon Committee (1994), Rakesh Mohan Committee (2001), Sam Pitroda Committee (2012) and Bibek Debroy Committee (2015).

36. Consider the following statements regarding Union territories of India:

1. The Parliament can’t make laws on any subject including the State List for union territories.

2. Legislative assembly of UTs can‘t make laws on any subject of the Concurrent List.

3. Presently there are 9 UT’s in India.

Which of the statements given above is/are incorrect?

A. 1 and 2 only

B. 2 and 3 only

C. 1 and 3 only

D. All of the above

Answer: D

Explanation: The Parliament can make laws on any subject of the three lists (including the State List) for the union territories. This power of Parliament also extends to Puducherry and Delhi, which have their own local legislatures. This means that the legislative power of Parliament for the union territories on subjects of the State List remains unaffected even after establishing a local legislature for them.

The legislative assembly of Puducherry can also make laws on any subject of the State List and the Concurrent List. Similarly, the legislative assembly of Delhi can make laws on any subject of the State List (except public order, police, and land) and the Concurrent List.

Presently there are 8 UTs in India after the creation of J&K and Ladakh and the merger of Daman and Diu, and Dadra and Nagar Haveli.

37. The National Air Quality Index mainly consists of:

1. Carbon dioxide

2. Carbon monoxide

3. Nitrogen dioxide

4. Sulfur dioxide

Page 41: PRELIMS TEST SERIES - 2020 - Sleepy Classes

www.YouTube.com/SleepyClasses

www.sleepyclasses.com/

5. Methane

Select the correct answer using the code given below:

A. 1, 2 and 3 only

B. 1, 4 and 5 only

C. 2, 3 and 4 only

D. 3, 4 and 5 only

Answer: C

Explanation:

The National Air Quality Index

(AQI), is ‘One Number- One Colour-One Description’ for the common man to judge the air quality within his vicinity. There are six AQI categories, namely Good, Satisfactory, Moderately polluted, Poor, Very Poor, and Severe.

The AQI considers eight pollutants, PM10, PM2.5, NO2, SO2, CO, O3, NH3, and Pb, for which short-term National Ambient Air Quality Standards are prescribed. National Air Quality Index (AQI) was launched on 17 October 2014.

38. Consider the following statements about Solid waste management techniques:

1. Pyrolysis is the process of burning

waste in large furnaces at high temperatures.

2. Incineration is a process of combustion in the absence of oxygen or the material burnt under controlled oxygen supply.

3. Composting is a biological process in which micro-organisms decompose organic waste in the presence of oxygen.

Which of the statements given above is/are incorrect?

A. 1 and 2 only

B. 2 and 3 only

C. 3 only

D. 1, 2 and 3 only

Answer: A

Explanation: The process of combustion in the absence of oxygen or the material burnt under a controlled atmosphere of oxygen is

called pyrolysis. It is an alternative to incineration. The gas and

liquid thus obtained can be used as fuels. It is Incineration, which includes the combustion of waste for recovering energy at high temperatures.

The process of burning waste in large furnaces at high temperatures is known as incineration. In these plants, the recyclable material is segregated and the rest of the material is burnt and ash is produced. Burning garbage is not a clean process as it produces tonnes of

toxic ash and pollutes the air and water.

Composting is a biological process in which microorganisms, mainly fungi, and bacteria, decompose degradable

organic waste into a humus like substance in the presence of oxygen. This finished product, which looks like soil, is high in carbon and nitrogen and is an excellent medium for growing plants.

39. Consider the following statements:

1. It is one of the clean coal technologies were coal is converted into synthesis gas, also called Syngas.

2. Syngas is a mixture of hydrogen, Sulphur dioxide, and Ozone.

3. India’s first coal gasification based fertilizer plant is to be set up in Karnataka.

Page 42: PRELIMS TEST SERIES - 2020 - Sleepy Classes

www.YouTube.com/SleepyClasses

www.sleepyclasses.com/

Which of the statements given above is/are correct about Syngas?

A. 1 only

B. 1 and 3 only

C. 2 and 3 only

D. 1, 2 and 3

Answer: A

Explanation: The gasification of coal is a method that can produce power, liquid fuels, chemicals and hydrogen. It is a clean coal technology and involves the process of converting coal into synthesis gas (also called syngas).

Syngas is a mixture of hydrogen, carbon monoxide, and carbon dioxide. The by-products of coal gasification include coke, coal tar, sulfur, ammonia, and fly ash, all having their own potential uses.

India’s first coal gasification based fertilizer plant is to be set up in

Talcher, Odisha. Carbon dioxide and ammonia produced from syngas are further reacted to produce urea.

40. Which of the following statements with respect to the Palm Oil industry are incorrect?

1. India is the world’s largest exporter of palm oil.

2. Indonesia and Malaysia contribute more than 85% of the world's Import of palm oil.

3. Kerala is the leading palm oil-producing state in India.

Select the correct code?

A. 1 and 2 only

B. 1 and 3 only

C. 2 and 3 only

D. 1, 2 and 3

Answer: D

Explanation:

India is the world’s largest importer

of palm oil, driving 23 percent of total global demand from plantations in Indonesia and Malaysia. Palm oil also contributes to around 70 percent of the total edible oils that are imported into the country.

Indonesia and Malaysia contribute almost 87% of the production of palm

oil. Indonesia produces 33 million tonnes of the commodity annually, while Malaysia 19.5 million tonnes. Other players are Thailand, Columbia, and Nigeria.

Andhra Pradesh is the leading palm oil-producing state in India contributing approximately 86% of the country’s production, followed by Kerala and Karnataka. Other palm oil-producing states include Orissa, Tamil Nadu, Goa, and Gujarat.

41. South Asia’s first cross border Petroleum Products Pipeline project has been signed between India and:

A. Bhutan

B. Nepal

C. Bangladesh

D. Sri Lanka

Answer: B

Explanation:

India and Nepal have jointly signed an agreement for the implementation of Cross border petroleum products

pipeline from Motihari in India to Amlekhgunj in Nepal. This will be the first such pipeline project in South Asia.

Page 43: PRELIMS TEST SERIES - 2020 - Sleepy Classes

www.YouTube.com/SleepyClasses

www.sleepyclasses.com/

The 69-km Motihari-Amlekhgunj pipeline, having a capacity of 2 million metric tons per annum, will provide cleaner petroleum products at an affordable cost to the people of Nepal.

42. Which of the following renewable sources of energy has the highest installed capacity in India?

A. Solar power

B. Wind power

C. Hydropower

D. Biopower

Answer: B

Explanation: India’s renewable

energy capacity has crossed the 80GW-mark, which includes 29.55 GW of solar energy and 36.37 GW wind power. The government has set an ambitious target of having 175 GW of clean energy capacity by 2022, including 100 GW solar and 60 GW of wind energy

43. Consider the following the statements

1. Climate forcings are factors in the climate system that either increase or decrease the effects to the climate system.

2. Negative forces such as excess greenhouse gases warm the earth.

3. Positive forces such as the effects of most aerosols and volcanic eruptions cool the earth.

Which of the above-mentioned statements is/are correct regarding Carbon Forcings?

A. 1 only

B. 1 and 2 only

C. 2 and 3 only

D. 1, 2 and 3

Answer: A

Explanation:Climate forcings are factors in the climate system that either increase or decrease the effects to the climate system. Positive forcing such as excess greenhouse gases warm the earth while negative forcing’s, such as the effects of most aerosols and volcanic eruptions, actually cool the earth.

44. The term Geotail is being frequently used in news now a day’s, Consider the

following statements about the same:

1. It is a region in space.

2. It exists as a result of the interactions between the Sun and the Moon.

3. Once every 29 days, the Moon traverses the geotail for about six days.

Which of the statements given above is/are incorrect?

A. 1 and 2 only

B. 2 only

C. 1 and 3 only

D. 1, 2 and 3

Answer: B

Explanation:

The geotail is a region in space that allows the best observations. The region exists as a result of the interactions between the Sun and Earth.

The Sun emits the solar wind, which is a continuous stream of charged particles. Since the Earth has a magnetic field, it obstructs the solar wind plasma.

This interaction results in the formation of a magnetic envelope around Earth. On the Earth side facing the Sun, the envelope is compressed into a region that is approximately three to four times the Earth's radius.

Page 44: PRELIMS TEST SERIES - 2020 - Sleepy Classes

www.YouTube.com/SleepyClasses

www.sleepyclasses.com/

On the opposite side, the envelope is stretched into a long tail, which extends beyond the orbit of the Moon. It is this tail that is called the geotail.

Once every 29 days, the Moon traverses the geotail for about six days. When Chandrayaan-2, which is orbiting the Moon, crosses the geotail, its instruments can study the properties of the geotail.

45. Consider the following statements about the Project Tiger in India:

1. Project Tiger is being implemented in more than half of Indian states.

2. As per the 2018 Tiger Census, Uttrakhand has the highest number of tiger populations in the country.

3. Out of all the Tiger reserves, the Bandhavgarh Tiger Reserve is the oldest one.

Which of the statements given above is/are correct?

A. 1 only

B. 2 and 3 only

C. 1 and 3 only

D. 3 only

Answer: A

Explanation:

Wild tigers are found in 18 of our States, viz. Andhra Pradesh, Arunachal Pradesh, Assam, Bihar, Chhattisgarh, Jharkhand, Karnataka, Kerala, Madhya Pradesh, Maharashtra, Mizoram, Odisha, Rajasthan, Tamil Nadu, Telangana, Uttarakhand, Uttar Pradesh and West Bengal.

Project Tiger is being implemented in these States in collaboration with the respective State Governments. With 526 tigers, Madhya Pradesh has

topped the state list in the tiger population under the 2018 Tiger

Census. Karnataka is the second ranker with 524 tigers

46. Consider the following statements about KUSUM scheme:

1. It is an initiative of the Ministry of Agriculture & Farmers' Welfare.

2. Under this scheme, farmers can avail of subsidies for the purchase of solar-powered pumps.

3. The main aim of this scheme is to install on-grid solar pumps only. Which of the above statements is/are correct with respect to KUSUM (Kisan Urja Suraksha evam Utthaan Mahabhiyan) scheme:

Select the correct option

A. 1 and 2 only

B. 2 only

C. 2 and 3 only

D. All of the above

Answer: B

Explanation:

'Kisan Urja Suraksha evam Utthaan

Mahabhiyan’ (KUSUM) is the full name for the scheme KUSUM. KUSUM scheme is launched on 1st February

2018 in the Union budget of 2018. It is a 4 Year scheme supervised by the Indian power ministry. Under this arrangement, the Central Govt. desires to assist as many farmers as possible to install new and improved solar

pumps on their farms.

Objective

To install 17.5 lakh off-grid and 10 lakh on-grid solar pumps and 10 gigawatts of solar power plant capacity in rural areas by 2022.

Page 45: PRELIMS TEST SERIES - 2020 - Sleepy Classes

www.YouTube.com/SleepyClasses

www.sleepyclasses.com/

To increase farmer income through the sale of surplus power, reduce electricity subsidy burden, and expand the distributed renewable energy capacity.

To provide farmers with advanced technology to generate power.

To enable a farmer to sell the

extra energy directly to the Power Supply Companies

47. Consider the following statements regarding the Emissions Gap Report 2019:

1. G20 nations collectively account for 78 per cent of all global greenhouse gas emissions.

2. The global GHG emission should fall by 7.6 per cent each year from 2020 to 2030 to meet the 1.5°C Paris climate agreement target.

3. This report is published by UNDP.

Which of the statements given above is/are correct?

A. 1 and 2 only

B. 1 and 3 only

C. 2 and 3 only

D. All of the above

Answer: A

Explanation: The Emission Gap is also called the commitment gap. It measures the gap between what we need to do and what we are doing to tackle climate change. It measures the pace at which emissions must be reduced to reach an emission low that would limit the temperature increase to 1.5°C.

48. Consider the following statements:

1. Its objective is to improve the livelihoods of forest-dependent communities in the Central Indian Highlands.

2. It is financed by the centre and states in varying proportions.

3. It is implemented under the National Mission for Green India. Which of the statements given above is/are correct with reference to Ecosystem Service Improvement Project :

Choose the correct option

A. 1 and 2 only

B. 3 only

C. 1 and 3 only

D. 1, 2 and 3

Answer: C

Explanation:

A Grant Agreement from the Global Environment Facility (GEF) of the World Bank for ―Ecosystem Service

Improvement Project‖ was signed by the Indian Government in the recent past. The size of Project is USD 24.64 million which entirely be financed by the World Bank out of its GEF Trust Fund. The project‘s duration is 05 years.

Ministry of Environment, Forest and Climate Change (MoEF&CC) will implement the Project in the States of Chhattisgarh and Madhya Pradesh through Indian Council of Forestry Research & Education under the National Mission for Green India (GIM).

The objective of the project is to strengthen the institutional capacity of the Departments of Forestry and Community Organisations to enhance forest ecosystem services and

Page 46: PRELIMS TEST SERIES - 2020 - Sleepy Classes

www.YouTube.com/SleepyClasses

www.sleepyclasses.com/

improve the livelihoods of forest-dependent communities in the

Central Indian Highlands.

The ecosystem services improvement project (ESIP) will support the goals of GIM by demonstrating models for adaptation based mitigation through sustained land and ecosystem management and livelihood benefits.

The ESIP project has the following components

Strengthen the capacity of government institutions in Forestry and Land management programs.

Investments for improving forest

quality in selected landscapes which include enhancing and restoring carbon stock in forest land.

Developing community-based models for sustainable utilisation of NTFP.

Scaling-up of sustainable land and ecosystem management in selected landscapes.

49. Consider the following statements with reference to Indian Rhino Vision 2020 (IRV 2020) programme:

1. It aims to attain a wild population of at least 3,000 greater one-horned rhino in

Assam by the year 2020.

2. It is the lone initiative of Assam‘s Forest Department in collaboration with Government of India.

3. It involves creating new habitats for Rhino by spreading them over different protected areas in all N-E States.

Which of the statements given above is/are correct?

A. 1 only

B. 2 and 3 only

C. 1 and 3 only

D. 1, 2 and 3

Answer: A

Explanation: Greater one-horned, or Indian, rhinoceros once roamed from Pakistan to the Indo-Burmese border, and in parts of Nepal, Bangladesh and Bhutan. But by the beginning of the 20th century, hunting and habitat loss had reduced the species to fewer than 200 individuals in northern India and Nepal. Indian Rhino

Vision 2020 is an ambitious effort to attain a wild population of at least 3,000 greater one-horned rhinos spread over seven protected areas in the Indian state of Assam by the year 2020.

Rhinos need to move to ecologically similar but distant areas to ensure species survival, according to the Indian Rhino Vision 2020 programme (IRV2020), a collaborative effort between various organisations, including the International Rhino Foundation, Assam‘s Forest Department, Bodoland Territorial Council, WWF-India, and the US Fish and Wildlife Service.

IRV2020 hopes to raise the number of rhinos in Assam to 3,000 by 2020 and spread them over seven of the state‘s protected areas: Kaziranga, Pobitora, Orang national park, Manas national park, Laokhowa wildlife

sanctuary, Burachapori wildlife sanctuary and DibruSaikhowa wildlife sanctuary.

50. The term “Common but Differentiated Responsibilities” is related with:

A. Climate Change Mitigation Efforts

B. Solving NPA crisis

C. center-state relation and Good Governance

D. Eliminating Single Use Plastic

Page 47: PRELIMS TEST SERIES - 2020 - Sleepy Classes

www.YouTube.com/SleepyClasses

www.sleepyclasses.com/

Answer: A

Explanation: Common but Differentiated

Responsibilities and Respective Capabilities (CBDR–RC) is a principle within the United Nations Framework Convention on Climate Change (UNFCCC) that acknowledges the different capabilities and differing responsibilities of individual countries in addressing climate change.

51. Bio Ethanol is being promoted now a day’s as an alternative source of fuel. Which of the following crops is/are the source for the same?

1. Sugarcane

2. Wheat

3. Sorghum

4. Tendu leaves

Select the correct answer using the code is given below:

A. 3 only

B. 1 and 2 only

C. 1 and 3 only

D. 1, 2 and 3

Answer: D

Explanation:

The principle fuel used as a petrol substitute for road transport vehicles is bioethanol. Bioethanol fuel is mainly produced by the sugar fermentation process, although it can also be

manufactured by the chemical process of reacting ethylene with steam.

It is produced from an agricultural products such as corn, sugarcane, potatoes, rice, beetroot and recently using grapes, banana, dates and other

wastes. There is also ongoing research and development into the use of

municipal solid wastes to produce ethanol fuel.

52. Fuel cells are being encouraged to be used as an alternative to traditional cell for generating electricity. Consider the following statements for the same:

1. It is a device that converts chemical potential energy into electrical energy.

2. It produces electricity in the form of Alternating Current.

3. It produces Water, Heat, and just a small fraction of CO2 as a by-product.

Which of the statements given above is/are incorrect?

A. 1 and 2 only

B. 2 and 3 only

C. 1 and 3 only

D. None of the above

Answer: B

Explanation: A fuel cell is a device that converts chemical potential energy (energy stored in molecular bonds) into electrical

energy. A PEM (Proton Exchange Membrane) cell uses hydrogen gas (H2) and oxygen gas (O2) as fuel. The products of the reaction in the cell are water, electricity, and heat. This is a big improvement over internal combustion engines, coal-burning power plants, and nuclear power plants, all of which produce harmful by-products. Since O2 is readily available in the atmosphere, we only need to supply the fuel cell with H2 which can come from an electrolysis process.

The fuel cell stack is the heart of a fuel cell power system. It generates electricity in the form of direct current

(DC) from electrochemical reactions that take place in the fuel cell. A single fuel cell produces less than 1 V, which is insufficient for most applications.

Page 48: PRELIMS TEST SERIES - 2020 - Sleepy Classes

www.YouTube.com/SleepyClasses

www.sleepyclasses.com/

Therefore, individual fuel cells are typically combined in series into a fuel cell stack. A typical fuel cell stack may consist of hundreds of fuel cells.

53. Consider the following statements:

1. It is a global partnership including governments and Indigenous people only. Aiming to reduce emissions from forest degradation.

2. The Global Environment Facility (GEF) makes a minimum financial contribution of $5 million to FCPF every year.

3. Its objective is to assist countries in their REDD+ efforts by providing them with financial and technical assistance. Which of the statements given above is/are correct With reference to Forest Carbon Partnership Facility?

Select the correct option

A. 3 only

B. 1 and 3 only

C. 2 and 3 only

D. 1, 2 and 3

Answer: A

Explanation: The Forest Carbon Partnership

Facility is a global partnership of governments, businesses, civil society, and Indigenous peoples focused on reducing emissions from deforestation and forest degradation.

The FCPF has two separate but complementary funding mechanisms — the Readiness Fund and the Carbon Fund- to achieve its strategic objectives. Both funds are underpinned by a multi-donor fund of governments and non-governmental entities, including private companies that make a minimum financial contribution of $5 million.

Objectives of the FCPF is to assist countries in their REDD+ efforts by providing them with financial and technical assistance in building their capacity to benefit from possible future systems of positive incentives for REDD+.

54. The bicuspid valve is located between of the following chambers of the heart?

A. Left Atrium and Left Ventricle

B. Right Atrium and Right Ventricle

C. Left Atrium and Right Atrium

D. Left Ventricle and Right Ventricle

Answer: A

Explanation:

The Mitral valve also called the bicuspid valve is the Left atria-ventricular valve which consists of two cusps. It prevents the reverse flow of oxygenated blood from the left ventricle to the left atrium

The tricuspid valve is located between the right atrium and right ventricle which consists of three cusps and it prevents the reverse flow of deoxygenated blood from the right ventricle to the right atrium.

Page 49: PRELIMS TEST SERIES - 2020 - Sleepy Classes

www.YouTube.com/SleepyClasses

www.sleepyclasses.com/

55. The government recently launched the I-STEM Portal. It is related with

A. Promoting judicious use of water

B. Intellectual property rights

C. Research and development facilities

D. Zero budget natural farming

Answer: C

Explanation:

Launched by the Nanoscience Centre of the Indian Institute of Science, Bengaluru, Indian Science Technology and Engineering facilities Map (I-STEM) portal is designed to be the gateway for researchers to locate a specific type of facility they need for their R&D work in India

Billed as ‘One Nation One Research Web Portal’ for the scientific community, the portal will link researchers and resources, hold a database of all the R&D facilities established in institutions around the country and enable their sharing in a transparent manner

56. Which of the following are categorised as dwarf planetsGoblin

1. Bennu

2. Ceres

3. Didymos

Select the correct option

A. 1 and 2

B. 3 and 4

C. 1 and 3

D. All of the above

Answer: C

Explanation:

Bennu and Didymos are the asteroids located between Mars and Jupiter.

Dwarf planet Ceres is the largest object in the asteroid belt between Mars and Jupiter and the only dwarf planet located in the inner solar system.

Goblin is an extremely distant dwarf planet which has redefined the outer reaches out the solar system

57. Which of the following are correct in the

context of X-rays?

1. Only gamma-rays carry more energy than the X-rays

2. X-rays do not penetrate the Earth’s atmosphere

Choose the correct option

A. 1 only

B. 2 only

C. Both 1 and 2

D. Neither 1 nor 2

Answer: C

Explanation:

Statement 1: X-rays are at the short wavelength, high energy end of the electromagnetic spectrum. Only

Page 50: PRELIMS TEST SERIES - 2020 - Sleepy Classes

www.YouTube.com/SleepyClasses

www.sleepyclasses.com/

gamma-rays carry more energy. It is convenient to describe x-rays in terms of the energy they carry, in units of thousands of electron volts (keV). X-rays have energies ranging from less than 1 keV to greater than 100 keV.

Statement 2: X-rays do not penetrate the Earth's atmosphere. Therefore they must be observed from a platform launched above most of our atmosphere.

58. Which of the following are humanoids that are deployed by various countries in space?

1. Fedor

2. Kirobo

3. Robonaut 2

Select the correct option

A. 1 and 3

B. 2 and 3

C. 1 and 3

D. 1, 2 and 3

Answer: D

Explanation:

Fedor (Final Experimental Demonstration Object Research) is a Russian, life-size humanoid robot that was sent to assist astronauts on the International Space Station in 2019. It is also known as Skybot F850

NASA sent up Robonaut 2 in 2011, a humanoid robot developed with General Motors that had a similar aim of working in high-risk environments.

In 2013, Japan sent up a small robot called Kirobo along with the ISS's first Japanese space commander. Developed with Toyota, it was able to hold

conversations — albeit only in Japanese.

Read about Vyommitra and Manav

59. Consider the following statements in the context of plants outside earth

1. Cotton, rapeseed, and potato seeds have sprouted on the moon

2. Such attempts to grow plants in the International Space Station have not been successful yet.

Select the correct option

A. 1 only

B. 2 only

C. Both 1 and 2

D. Neither 1 nor 2

Answer: A

Explanation:

Statement 1 is correct as the Chinese space agency has confirmed that the Change 4 lunar probe carried cotton, rapeseed, potato,

arabidopsis, yeast, and fruit flies. These six components behaved as producers, consumers, and decomposers. Cotton seeds, rapeseed, and potato seeds have sprouted. This is the first time humans have done biological growth experiments on the lunar surface.

Statement 2 is incorrect because planet has previously been grown at the International Space Station.

60. GPS Aided GEO Augmented Navigation (GAGAN) is used in which of the following

1. Aviation

2. Deep-sea fishing

3. Railways

Page 51: PRELIMS TEST SERIES - 2020 - Sleepy Classes

www.YouTube.com/SleepyClasses

www.sleepyclasses.com/

Choose the correct option

A. 1 and 2 only

B. 1 and 3 only

C. 1 only

D. All of the above

Answer: D

Explanation:

GPS Aided GEO Augmented

Navigation (GAGAN), developed by the Airports Authority of India (AAI) and ISRO is a step towards initial Satellite-based Navigation Services in India. It is a system to improve the accuracy of a global navigation satellite system (GNSS) receiver by providing reference signals.

GAGAN is the first Satellite-Based Augmentation System in the world that has been certified for approach with vertical guidance operating in the equatorial ionospheric region.

GAGAN covers the area from Africa to Australia and has expansion capability for seamless navigation services across the region.

In addition to aviation, where it is used for accurate landing of the aircraft, GEMINI device is used as a satellite-based advisory service for deep-sea fishermen.

Real-time Train Information System (RTIS) monitors the speed and movement of trains uses GAGAN to transmit signals.

61. Goldschmidtite is

A. a new gold ore

B. the mineral from Earth’s mantle

C. a plant that takes in oxygen

D. a highly efficient nuclear fuel

Answer: B

Explanation:

It is a new mineral that is discovered inside a diamond from South Africa. It has an unusual chemical signature for a mineral from Earth’s mantle.

While the mantle is dominated by elements such as magnesium and iron, goldschmidtite has high concentrations of niobium, potassium and the rare earth elements lanthanum and cerium

62. Which of the following statement(s) is/are correct in the context of Lymphatic

Filariasis

1. India has launched a triple-drug therapy regimen to eliminate lymphatic filariasis from the country by 2021

2. Short-term tourists are at comparatively lower risk than the local people

3. It is only spread by Culex mosquitoes

Select the correct option

A. 1 and 3 only

B. 2 and 3 only

C. 1 and 2 only

D. 1, 2 and 3

Answer: C

Explanation:

Statement 1 is correct as India has

launched a triple-drug therapy (Ivermectin, Albendazole, and Diethylcarbamazine) regimen to achieve the ambitious target of eliminating lymphatic filariasis from the country by 2021.

Statement 2 is correct because Lymphatic Filariasis happens with

Page 52: PRELIMS TEST SERIES - 2020 - Sleepy Classes

www.YouTube.com/SleepyClasses

www.sleepyclasses.com/

multiple mosquito bites over several months. Hence short-term tourists are at very low risk.

Statement 3 is incorrect because Lymphatic filariasis is transmitted by different types of mosquitoes for example by the Culex mosquito, widespread across urban and semi-urban areas, Anopheles, mainly found in rural areas, and Aedes, mainly in endemic islands in the Pacific.

63. ‘Abhyas’ has been in news recently. It relates to

A. disaster management mock drill

B. preparing poor students for competitive exams

C. preparatory portal for unforeseen health emergencies

D. drone by DRDO

Answer: D

Explanation:

Abhyas, a High-speed Expendable Aerial Target (HEAT) is a drone. It is designed to offer a realistic threat scenario for the practice of weapon systems. But it is much more than an aerial target. Besides training purposes, it can be used for multiple things. It is the country's first locally developed system

It has multiple applications. Its Radar Cross Section is increased 50 times to imitate a fighter jet. It also has the potential to be converted into a high-speed subsonic missile. It can act as a decoy and also function as a jammer platform. It is capable to carry warheads. But in its present form, Abhyas is not meant for that. But in the near future, it can be used depending upon your requirement.

64. Which of the following fall under the category of anti-tank missiles

1. Nag

2. Spike

3. Strum Ataka

4. Javelin

Select the correct option

A. 1 and 2 only

B. 1, 2 and 3 only

C. 1, 2 and 4 only

D. 1, 2, 3 and 4

Answer: D

Explanation: An anti-tank missile, anti-tank guided missile (ATGM), anti-armour guided missile, or anti-tank guided weapon (ATGW) is a missile that is created to destroy vehicles that are heavily armoured.

Nag is India’s indigenous anti-tank guided missile

Spike an Israeli-made ATGM developed by Rafael Advanced Defence Systems. India procured these missiles

India has signed a deal to acquire ‘Strum Ataka’ anti-tank missile from Russia in 2019 US offered Javelin Anti-tank Guided Missiles through the Defense Technology and Trade Initiative between India and US

65. Bhashan Char Island, sometimes in news

is situated at the mouth of river

A. Meghna

B. Irrawaddy

C. Kaladan

D. Padma

Answer: A

Page 53: PRELIMS TEST SERIES - 2020 - Sleepy Classes

www.YouTube.com/SleepyClasses

www.sleepyclasses.com/

Explanation:

Approximately 6000-7000 Rohingya refugees have agreed to be relocated to the newly built camp on the Bhashan Char Island.

Bhashan Char also known as Thengar Char, is an uninhabited island around 30 kilometers east of Hatiya island in South East Bangladesh. Hatiya has a population of 600,000. Bhashan Char island was formed about two decades ago on the mouth of river Meghna.

66. Beijing Declaration and Platform for Action, often seen in the news, is (UPSC CSE 2015)

A. a strategy to tackle regional terrorism, an outcome of a meeting of SCO

B. a plan of action for sustainable economic growth in the Asia-Pacific region, an outcome of the deliberations of the Asia-Pacific Forum

C. an agenda for women’s empowerment, an outcome of a World Conference convened by the United Nations

D. a strategy to combat wildlife trafficking, a declaration of the East Asia Summit

Answer: C

Explanation:

The 1995 Beijing Declaration and Platform for Action remains the world’s most progressive blueprint for advancing gender equality worldwide.

On International Women’s Day 2020, the World Health Organization, United Nations University - International Institute for Global Health and the British Medical Journal launched a special series marking ‘Beijing+25’.

It shines a spotlight on health, education, environment, work and other critical foundations of life as they empower or restrict the rights and well-being of girls and women

67. In the wake of the COVID-19 pandemic,

which of the following has decided to issue Corona bonds?

A. IMF

B. World Bank

C. European Union

D. RCEP

Answer: C

Explanation:

Corona bonds are joint debt issued to member states of the EU. The funds would be common and would come from the European Investment Bank.

This would be mutualised debt, taken collectively by all member states of the European Union.

68. Which one of the following is the best

description of ‘INS Astradharini’, that was in the news recently? (UPSC CSE 2016)

A. Amphibious warfare ship

Page 54: PRELIMS TEST SERIES - 2020 - Sleepy Classes

www.YouTube.com/SleepyClasses

www.sleepyclasses.com/

B. Nuclear-powered submarine

C. Torpedo launch and recovery vessel

D. Nuclear-powered aircraft carrier

Answer: C

Explanation: Hindustan Shipyard Ltd achieved another milestone in ship repair capability by completing the maiden refit of INS Astradharini, a torpedo launch and recovery vessel of Indian Navy.

69. What is IndARC?

A. Air missile defense system to be developed by India

B. Device to treat cancer

C. Neutrino observatory

D. Indigenous particle accelerator developed by India

Answer: A

Explanation:

The ESSO-National Centre for Antarctic and Ocean Research (NCAOR) and the ESSO-National Institute of Ocean Technology (NIOT) successfully deployed IndARC, the country’s first multi-sensor moored observatory in the Kongsfjorden fjord of the Arctic, roughly halfway between Norway and the North Pole.

The IndARC observatory is an attempt to collect continuous data from depths very close to the water surface as well as at different discrete depths. The data acquired would be of vital importance to the Indian climate researchers as well as to the international fraternity.

In addition to providing for an increased understanding of the response of the Arctic to climatic variabilities, the data would also provide a good handle in our understanding of the Arctic processes

and their influence on the Indian monsoon system through climate modeling studies.

70. Which of the following statements are correct with respect to Neolithic period?

1. Mud brick houses were built instead of grass huts during Neolithic period.

2. Evidence of pottery is absent from the Neolithic culture.

3. The people of Neolithic Age used clothes made of cotton and wool.

4. Domestication of animals started during Neolithic culture.

Select the correct code.

A. 1 and 3 only

B. 2 and 4 only

C. 2 and 3 only

D. 1 and 4 only

Answer: D

Explanation:

During Neolithic age, wheels were used to make pottery. Pottery was used for cooking as well as storage of food grains.

Domestication of animals, horticulture and primitive cultivation started during Mesolithic period itself. However, during Neolithic period, domestication of sheep, goats and cattle was widely prevalent. Cattle were used for cultivation and for transport.

71. Consider the following statements with reference to the condition of women in Rig Vedic society.

1. Women were not allowed to wear garments made of cotton.

Page 55: PRELIMS TEST SERIES - 2020 - Sleepy Classes

www.YouTube.com/SleepyClasses

www.sleepyclasses.com/

2. Child marriages were banned and the practice of sati was absent.

3. Women were not given equal opportunities as men for their spiritual and intellectual development.

Which of the above statements are correct?

A. 1 only

B. 1 and 3 only

C. 2 only

D. 3 only

Answer: C

Explanation:

The Rig Vedic society was patriarchal. But, the condition of women was much better in this period that in the Later vedic age.

Women were given equal opportunities as men for their spiritual and intellectual development. There were women poets like Apala, Viswavara, Ghosa and Lopamudra during the Rig Vedic period.

Women could even attend the popular assemblies. There was no child marriage and the practice of sati was absent.

Men and women wore upper and lower garments made of cotton and wool. A variety of ornaments were used by both men and women.

72. Senni, Valavan and Killi were famous titles assumed by

A. Pandya Kings

B. Temple priests

C. Chola Kings

D. Chera kings

Answer: C

Explanation:

Chola kings assumed the titles like Senni, Valavan and Killi.

Pandya kings: Thennavar and Minavar.

Chera kings: Vanavaramban, Vanavan, Kuttuvan, Irumporai and Villavar

73. Which of the following statements are correct with reference to the Fa Hien’s account of India?

1. His account was a portrayal of political affairs of Indian kingdoms.

2. He refers to the Gangetic valley as the

‘land of Brahmanism’.

3. He indicates the importance of the seven precious substances for Buddhist worship.

Select the correct answer using the codes below.

A. 1 and 3 only.

B. 2 only.

C. 3 only.

D. 2 and 3 only.

Answer: D

Explanation:

Between 399 and 414 CE, the Chinese monk Fa Hien undertook a trip via Central Asia to India seeking better copies of Buddhist books than were currently available in China.

He was not interested in political affairs. His interest was primarily religion.

According to him, Buddhism was in a flourishing condition in the north-western India but in the Gangetic valley it was in a state of neglect. He refers to the Gangetic valley as the ‘land of Brahmanism’.

Page 56: PRELIMS TEST SERIES - 2020 - Sleepy Classes

www.YouTube.com/SleepyClasses

www.sleepyclasses.com/

He indicates clearly the importance of the seven precious substances for Buddhist worship, the widespread practice of stupa veneration, and his aquaintance with several of the jataka tales about the previous lives of the Buddha Sakyamuni, tales which are illustrated in the paintings at the Dunhuang caves.

According to the Lotus Sutra, the seven are gold, silver, lapis lazuli, seashell, agate, pearl, and carnelian.

74. ‘Hiranya-garbha’ sacrifice was done during Rashtrakuta rule. It was done in

order to

A. Declare war.

B. Convert the caste of an individual to Kshatriya.

C. Celebrate the birthday of king.

D. None of the above.

Answer: B

Explanation:

This ritual was performed with the help of Brahmanas, it was thought to lead to the “rebirth” of the sacrificer as a Kshatriya, even if he was not one by birth.

In the mid-eighth century, Dantidurga, a Rashtrakuta chief, overthrew his Chalukya overlord and performed a ritual called hiranya-garbha (literally, the golden womb).

75. Consider the following statements with respect to Amir Khusrau.

1. He was a mystic and a spiritual disciple of Nizamuddin Auliya of Delhi.

2. His Khaliq-e-bari is about the achievements of Khilji rulers.

Which of the above statements are correct?

A. 1 only.

B. 2 only.

C. Both 1 and 2.

D. None of the above.

Answer: A

Explanation:

He wrote poetry primarily in Persian,

but also in Hindavi.

He also introduced the ghazal style of song into India, both of which still exist widely in India and Pakistan.

He also wrote a war ballad in Punjabi. In addition, he spoke Arabic and Sanskrit.

His Khaliq-e-bari is known as the oldest printed dictionary of the world which deals with Hindi and Persian words.

76. The fourth Buddhist Council was convened in Kashmir by Kanishka under

the chairmanship of

A. Vasumitra

B. Moggaliputta Tissa

C. Aśvaghoṣ a

D. Mahakasapa

Answer: A

Explanation: The fourth Buddhist council was held in 72 AD at Kundalvana, Kashmir. It was

presided by Vasumitra, while Asvaghosa was his deputy. The council was held under the patronage of Kushan King Kanishka of Kushan Empire.

77. During 16th century, “Kunjali Marakkar” was the title given to

Page 57: PRELIMS TEST SERIES - 2020 - Sleepy Classes

www.YouTube.com/SleepyClasses

www.sleepyclasses.com/

A. Royal poets of southern kingdoms.

B. Naval chief of the Zamorin

C. Village heads who were also landowners

D. Maratha’s army commander.

Answer: B

Explanation:

The Marakkars are credited with organizing the first naval defence of the Indian coast, to be later succeeded in the 18th century by the Maratha Sarkhel Kanhoji Angre.

There were four major Kunhalis who played a part in the Zamorin's naval wars with the Portuguese from 1502 to 1600. Of the four Marakkars, Kunjali Marakkar II is the most famous.

78. What were the important provisions in the Regulating act of 1773?

1. Provision was made in the Act for the establishment of a Supreme Court at Bombay.

2. A council of four members was appointed to assist the Governor-General.

3. The tenure of Governor-General of Fort William was extended from two year to four years.

Select the correct answer using the codes below.

A. 1 and 2 only.

B. 3 only

C. 2 and 3 only.

D. 2 only

Answer: D

Explanation:

It provided for the establishment of a Supreme Court at Calcutta (1774) with one Chief Justice and three other Judges. In 1774, the Supreme Court was established by a Royal Charter.

The Governor of Bengal was styled the Governor-General of Fort William whose tenure of office was for a period of five years.

A council of four members was appointed to assist the Governor-General. A council of four members was appointed to assist the Governor-General.

79. Consider the following statements with reference to Khilafat Movement.

1. The chief cause of the Khilafat Movement was the defeat of Turkey in the First World War.

2. Chittaranjan Das was associated with this movement.

Which of the above statements is/are correct?

A. 1 only

B. 2 only

C. Both 1 and 2

D. None of the above.

Answer: C

Explanation:

The chief cause of the Khilafat Movement was the defeat of Turkey in the First World War.

The harsh terms of the Treaty of Sevres (1920) was felt by the Muslims as a great insult to them.

The whole movement was based on the Muslim belief that the Caliph (the

Sultan of Turkey) was the religious head of the Muslims all over the world.

Page 58: PRELIMS TEST SERIES - 2020 - Sleepy Classes

www.YouTube.com/SleepyClasses

www.sleepyclasses.com/

The Khilafat Committee in Bengal included Mohmmad Akram Khan, Manruzzaman Islamabadi, Mujibur Rahman Khan and Chittaranjan Das.

80. The governor-General who convinced the ruler of Kashmir, Ranjit Singh to sign the Indus Navigation Treaty was

A. Lord William Bentinck

B. Lord Amherst

C. Lord Hastings

D. Lord Auckland

Answer: A

Explanation:

Lord William Bentinck was the first Governor-General to visualise a Russian threat to India.

Hence, he was eager to negotiate friendly relations both with the ruler of Punjab, Maharajah Ranjit Singh and also with the Amirs of Sind.

His earnest desire was that Afghanistan should be made a buffer state between India and any possible invader.

The meeting of Bentinck and Ranjit Singh on 25 October, 1831 at Rupar on the bank of the river Sutlej amidst show and splendor.

The Governor-General was successful in winning the friendship of Ranjit Singh and the Indus Navigation Treaty was concluded between them.

81. Which of the following statements are correct with reference to the Partition of Bengal (1905)?

A. “Surat Split” was also the cause of partition.

B. New provinces of East Bengal and Assam were created after the partition of Bengal.

C. Both 1 and 2.

D. None of the above.

Answer: B

Explanation:

The partition of Bengal in 1905 provided a spark for the rise of extremism in the Indian National Movement.

The new province of Eastern Bengal and Assam included the whole of Assam and the Dacca, Rajshahi and Chittagong divisions of Bengal with headquarters at Dacca.

Though Curzon justified his action on administrative lines, partition divided the Hindus and Muslims in Bengal. This led to the anti-partition agitation all over the country.

“Surat split” happened in 1907 at Surat session which was due to differences between Moderates and Extremists.

82. Consider the following statements with reference to the Factory Act of 1881.

1. It was introduced by Lord Ripon to

improve the service condition of the factory workers in India.

2. It categorised organized and unorganized industries in India.

3. Following this act, a Factory Commission was appointed in 1885.

Which of the above statements is/are correct?

A. 1 and 2 only.

B. 2 only.

C. 1 and 3 only.

D. 1, 2 and 3.

Page 59: PRELIMS TEST SERIES - 2020 - Sleepy Classes

www.YouTube.com/SleepyClasses

www.sleepyclasses.com/

Answer: C

Explanation: Features of Factory Act of 1881

The Act banned the appointment of children below the age of seven in factories.

It reduced the working hours for children.

It made compulsory for all dangerous machines in the factories to be properly

fenced to ensure security to the workers.

83. Vedaranyam Salt Satyagraha March was undertaken by who among the following?

A. M.K. Gandhi

B. Periyar E.V.R.

C. Rajagopalachari

D. Sarojini Naidu

Answer: C

Explanation:

As part of the Civil Disobedience Movement, Gandhi launched the Salt Satyagraha. He undertook the Dandi March in 1930.

C. Rajagopalachari became the TNCC president in April 1930. Authorized by TNCC and AICC to direct the Salt Satyagraha in Tamil Nadu Rajaji undertook the famous Vedaranyam Salt Satyagraha March.

Periyar E.V.R. launched the Vaikom Satyagraha in Kerala against the practice of social segregation.

84. Consider the following statements

1. Dyarchy was introduced in the provinces. Provincial subjects were divided into “Reserved Subjects” and “Transferred Subjects”.

2. A bicameral (Two Chambers) legislature was set up at the centre. It consisted of the Council of States and the Legislative Assembly.

The above statements refers to

A. Montague-Chelmsford Reforms of 1919

B. Minto- Morley Reforms of 1909

C. Indian Councils Act of 1892

D. The Government of India Act of 1935

Answer: A

Explanation: Montague-Chelmsford Reforms of 1919

The main features of the Act were:

Dyarchy was introduced in the provinces. Provincial subjects were divided into “Reserved Subjects” and “Transferred Subjects”. The Reserved subjects were to be administered by the Governor and his Executive Council. The Transferred subjects by the Governor and his ministers.

A bicameral (Two Chambers) legislature was set up at the centre. It consisted of the Council of States and the Legislative Assembly.

The salaries of the Secretary of State for India and his assistants were to be paid out of the British revenues. So far, they were paid out of the Indian revenues.

A High Commissioner for India at London was appointed.

The most important defect in this Act was the division of powers under the system of Dyarchy in the provinces.

85. During whose tenure, following events occurred?

1. Poona Pact

2. 2nd and 3rd Round Table Conference

Page 60: PRELIMS TEST SERIES - 2020 - Sleepy Classes

www.YouTube.com/SleepyClasses

www.sleepyclasses.com/

3. Government of India Act 1935

Select the Correct Option

A. Lord Mayo

B. Lord Irwin

C. Lord Willingdon

D. Lord Linlithgow

Answer: C

Explanation: Lord Willingdon was the

Viceroy of India from 1931 to 1936

86. The social base of the Swadeshi movement included which of the following groups?

1. Zamindari

2. Students

3. Women

4. Muslim peasantry

Choose the correct option:

A. 2 and 3 only

B. 2,3 and 4 only

C. 2 and 4 only

D. 1,2 and 3 only

Answer: D

Explanation:

The social base of the Swadeshi movement expanded to include certain sections of the zamindari, the students, the women, and the lower middle

classes in cities and towns.

An attempt was also made to give political expression to economic grievances of the working class by organising strikes.

But the movement was not able to garner support of the Muslims, especially the Muslim peasantry,

because of a conscious government policy of divide and rule helped by overlap of class and community at places.

87. Bosphorus Strait is between:

A. Caribbean Sea and North Atlantic

B. Gulf of Aden and the Red Sea

C. Gulf of Oman and the Persian Gulf

D. Black Sea to Sea of Marmara

Answer: D

Explanation:

Caribbean Sea and North Atlantic: Jamaica Channel

Gulf of Aden and the Red Sea: Bab-el-Mandeb

Gulf of Oman and the Persian Gulf: Hormuz Strait

Black Sea to Sea of Marmara: Bosphorus Strait

88. The brahmaputra ,Irrawady,and Mekong rivers originate in Tibet and flow through narrow and parallel mountain ranges in their upper reaches .Of these rivers , Brahmaputra makes a “U” turn in its

course to flow into India. This “U” turn is due to:

A. Uplift of folded Himalayan series

B. Syntaxial bending of Geologically young Himalyas

C. Geo Tectonic Disturbance in the tertiary folded mountain chains

D. Both (a) and (b) above

Answer: B

Explanation:

Page 61: PRELIMS TEST SERIES - 2020 - Sleepy Classes

www.YouTube.com/SleepyClasses

www.sleepyclasses.com/

Syntaxical bends are generally deep knee-bend flexures which occured at the extreme ends of west and east of the Himalayas due to southward turn of the mountain chain.

The western syntaxial bend is near Nanga Parwat.

The eastern syntaxial bend is at Namcha Barwa where mountains take southward turn.

So, these gently arching ranges of the Himalayan mountains on their Western and Eastern extremities sharply bent southward in deep Knee-bend flexures are called syntaxial bends

89. Identify the drainage pattern is made by the Ganga River system among the following:

A. Trellis Drainage Pattern

B. Angular Drainage Pattern

C. Rectangular Drainage Pattern

D. Radial Drainage Pattern

Answer: A

Explanation:

Dendritic Drainage Pattern: When the river channel follows the slope of the terrain, it develops dendritic pattern. The stream and its tributaries resemble the branches of a tree. Hence, it is

called dendritic pattern. Most of the rivers of the Indo-Gangetic Plains are of dendritic.

Ganga river system forms Dendritic pattern

90. Recently, a Dutch government’s proposal to build two mega dams to separate the North Sea from the Atlantic Sea, is being hailed as a future solution to defend

northern Europe from rising sea levels. The project is referred to as “ NEED- Northern European Enclosure Dam”.

Which one among the following is marked incorrectly in context with the same?

1. France and England: NEED-North

2. Scotland and Norway: NEED-South

Select the Correct Option

A. Only 1

B. Only 2

C. Both 1 and 2

D. Neither 1 nor 2

Answer: C

Explanation:

Recently, a Dutch government’s proposal to build two mega dams to separate the North Sea from the Atlantic Sea, is being hailed as a future solution to defend northern Europe from rising sea levels. Northern European Enclosure Dam (NEED) is a

Page 62: PRELIMS TEST SERIES - 2020 - Sleepy Classes

www.YouTube.com/SleepyClasses

www.sleepyclasses.com/

set of two proposed dams in the English Channel between: o Scotland and Norway: NEED-North France and England: NEED-South .

It would cut off the North and Baltic Seas from the Atlantic Ocean. When completed, it would be the longest dam in the world. o NEED would lock Europe's four busiest ports—Rotterdam, Antwerp, Bremerhaven, Hamburg—behind a huge dam. New harbours would need to be built on the exterior of the dams to accommodate the volume of traffic to the interior ports.

91. “Room of the River Project” has been in news recently.

Find the incorrect statements in context with the same:

A. The origin of the project can be traced back to Japan.

B. It is to be replicated in Kerala’s Kuttanad as a flagship project

C. It is a flood mitigation initiative which is aimed at protecting areas adjoining rivers from routine flooding and improving water management systems in delta regions.

D. None of the above.

Answer: A

Explanation:

One of the flagship projects of the Netherland government, “Room of the river” is to be replicated in Kerala’s Kuttanad, which remained submerged for weeks during Kerala floods.

It is a flood mitigation initiative which is aimed at protecting areas adjoining rivers from routine flooding and improving water management systems in delta regions.

Key concept behind the project:

To provide more space for the water body so that it can manage extraordinary high-water levels during floods.

The project involves tailor-made solutions for each River.

Important measures of the project are:

o lowering the flood plain,

o strengthening and relocation of dykes,

o reducing the height of the groynes ( a rigid hydraulic structure built either from the shore or river bank to dissipate the wave energy or to protect the banks from erosion).

o increasing the depth of the side channels and

o removing obstacles.

It will also improve the surroundings of the river banks through fountains and panoramic decks, for this the landscapes are altered in a way that they turn into natural sponges which can accommodate excess water during floods.

92. “Shannon-Weiner Index” has been in news recently.

Identify the incorrect statements in context with the same?

1. It has recently been used in India State of the Forest Report 2019.

2. The index is used for measuring species richness and abundance among various habitats.

Select the Correct Option

A. Only 1

B. Only 2

Page 63: PRELIMS TEST SERIES - 2020 - Sleepy Classes

www.YouTube.com/SleepyClasses

www.sleepyclasses.com/

C. Both 1 and 2

D. Neither 1 nor 2

Answer: D

Explanation:

India is one of the 17 mega diverse countries in the world as per the Shannon-Weiner Index used in India State of the Forest Report 2019. The index is used for measuring species

richness and abundance among various habitats.

The Index shows that o Tropical Evergreen forest is high in Karnataka followed by Kerala. o Semi-evergreen

forest are high in Karnataka. o Tropical moist deciduous forests cover is high in Arunachal Pradesh, Karnataka and Maharashtra. o Tropical dry deciduous forest is high in Arunachal Pradesh o Tropical Littoral and swamp forests are high in UP and o Tropical thorn forests are high in Andhra Pradesh.

93. As per Economic Survey 2020; find the

correct statements:

1. The agricultural credit flow target for 2019-20 has been fixed at Rs 13.5 lakh crore.

2. It is observed that credit is low in the Southern states whereas it is highest in North Eastern, Hilly and Eastern States

Select the Correct Option

A. Only 1

B. Only 2

C. Both 1 and 2

D. Neither 1 nor 2

Answer: A

Explanation: Agricultural Credit:

The agricultural credit flow target for 2019-20 has been fixed at Rs 13.5 lakh crore.

Skewed Regional Distribution: It is observed that credit is low in North Eastern, Hilly and Eastern States. Whereas it is highest in the Southern states. (Kerala, Andhra Pradesh and TN)

94. “SDG India Index 2019” is frequently in news.

Find the correct statements in context with the same:

1. It is more comprehensive than SDG India Index 2018 and highlights the progress being made by the States/UTs on a wider set of 100 indicators spread across 16 goals

2. India’s score in the composite SDG India Index 2019 has improved from 57 in 2018 to 60 in 2019.

3. The goals that demand special attention are – 2 (Zero Hunger) and 5 (Gender Equality).

Select the Correct Option

A. 1 and 2 only

B. 2 and 3 only

C. 1 and 3 only

D. All of the above

Answer: D

Explanation: SDG India Index 2019

It is more comprehensive than SDG India Index 2018 and highlights the progress being made by the States/UTs on a wider set of 100 indicators spread across 16 goals.

The SDG score varies from 0 to 100. A score of 100 implies that the States/UTs have achieved the targets set for 2030; a

Page 64: PRELIMS TEST SERIES - 2020 - Sleepy Classes

www.YouTube.com/SleepyClasses

www.sleepyclasses.com/

score of 0 implies that the particular State/UT is at the bottom of the table.

States with scores equal to/greater than 65 are considered as Front-Runners; in the range of 50-64 as Performers and as Aspirants if the score is less than 50.

India’s score in the composite SDG India Index 2019 has improved from 57 in 2018 to 60 in 2019 largely driven by commendable country wide performance in five SDG goals – 6 (Clean Water and Sanitation), 7 (Affordable and Clean Energy), 9 (Industry, Innovation and Infrastructure), 15 (Life on Land) and 16 (Peace, Justice and Social Institutions) - where India has scored between 65 and 99.

The goals that demand special attention are – 2 (Zero Hunger) and 5 (Gender Equality) – where the overall country score is below 50.

As per the Index, Kerala, Himachal Pradesh, Tamil Nadu, Andhra Pradesh, Telangana, Karnataka, Goa, Sikkim, Chandigarh and Puducherry are the front runners while none of the States/ UTs fall in the Aspirant category.

95. Which of the following is matched incorrectly?

1. Chang’e-4: China

2. Hayabusa2: Japan

3. SnowEx : Russia

4. Voyager-2 : NASA

Select the Correct Option

A. Only 1

B. Only 2

C. Only 3

D. Only 4

Answer: C

Explanation:

Chang’e-4

It is part of the second phase of China’s lunar programme. It is the first mission to land on the far side of the Moon. It landed at the South Pole- Aitken Basin.

Hayabusa2

It is Japan’s asteroid-sampling spacecraft which landed at asteroid Ryugu in June of 2018 and recently began its journey back to earth.

SnowEx

It is a five-year program launched in 2016-17 by NASA to understand how much water is contained in each winter’s snowfall and how much will be available when it melts in spring.

Voyager-2

Like Voyager 1, Voyager 2 was designed to find and study the edge of our solar system. It is the only spacecraft to study all four of the solar system's giant planets-Jupiter, Saturn, Uranus and Neptune at close range.

96. Identify the tectonic Lakes among the following:

1. Wular Lake

2. Lake Baikal

3. Caspian Sea

Select the Correct Option

A. 2 only

B. 3 only

C. 2 and 3 only

D. 1 ,2 and 3

Answer: D

Page 65: PRELIMS TEST SERIES - 2020 - Sleepy Classes

www.YouTube.com/SleepyClasses

www.sleepyclasses.com/

Explanation:

Tectonic lakes are lakes formed by the deformation and resulting lateral and vertical movements of the Earth's crust.

These movements include faulting, tilting, folding, and warping. Some of the well-known and largest lakes on Earth are rift lakes occupying rift valleys, e.g. Central African Rift lakes,Caspian Sea, the Sea of Aral and Lake Baikal.

Wular lake (Jammu & Kashmir) is the largest freshwater lake in India. It was formed by tectonic activity.

97. Jet streams are a narrow variable band of very strong predominantly westerly air currents encircling the globe several miles above the earth. Which of the following are incorrect in context with the same?

1. Jet streams help in maintenance of latitudinal heat balance by mass exchange of air.

2. Jet streams are used by aviators if they have to fly in the direction of the flow of the jet streams, and avoid them when flying in opposite direction.

Select the Correct Option

A. Only 1

B. Only 2

C. Both 1 and 2

D. Neither 1 nor 2

Answer: D

Explanation: Influence of Jet Streams

Jet streams help in maintenance of latitudinal heat balance by mass exchange of air.

PFJ influence the mid-latitude weather disturbances. Usually there are severe

storms when jet streams interfere with surface wind systems.

Jet streams also influence the path of temperate cyclones. They have an influence on distribution of precipitation by the temperate cyclones.

Sub-tropical jet stream and some temporary jet streams together influence Indian Monsoon patterns. (more about this while studying India Monsoons in Indian geography)

Jet streams also exercise an influence on movement of air masses which may cause prolonged drought or flood conditions.

Jet streams are used by aviators if they have to fly in the direction of the flow of the jet streams, and avoid them when flying in opposite direction.

Jet streams can also cause a bumpy flight, because the jet stream is sometimes unpredictable and can cause sudden movement, even when the weather looks calm and clear.

During volcanic eruptions plumes of volcanic ash have a tendency to get sucked into the same jet stream that airplanes use for travel.

98. Which of the following are incorrect with respect to Ecocline:

1. Thermocline -Based on difference in water temperature

2. Chemocline - Based on difference in density

3. Halocline - Based on difference in water salinity.

4. Pycnocline - Based on difference in chemical concentration

Select the Correct Option

Page 66: PRELIMS TEST SERIES - 2020 - Sleepy Classes

www.YouTube.com/SleepyClasses

www.sleepyclasses.com/

A. 1 and 3 only

B. 2 and 4 only

C. 1,3 and 4

D. 2 only

Answer: B

Explanation: Ecocline or simply cline is in which series of biocommunities display a continuous gradient. Example - Thermocline is gradient based on temperature. Similarly, chemocline on chemical concentration, pycnocline on density and halocline on salinity.

99. Which of the following passes connect Chumbi Valley to Sikkim?

1. Nathu La

2. Jalep La

3. Shipki La

4. Bomdi La

Select the Correct Option

A. 1 Only

B. 1 and 2 only

C. 1, 2 and 3 only

D. 1, 2, 3 and 4

Answer: B

Explanation: The Chumbi Valley is connected to Sikkim to the southwest via the mountain passes of Nathu La and Jelep La.

100. Identify the correct statements in context with “Seed Replacement Ratio”:

1. Seed Replacement Ratio is a measure of how much of the total cropped area was sown with certified seeds in comparison to farm saved seeds.

2. It represents the access of farmers to quality seed and inversely proportional to productivity of farming.

Select the Correct Option

A. Only 1

B. Only 2

C. Both 1 and 2

D. Neither 1 nor 2

Answer: A

Explanation: Seed Replacement Ratio (SRR)

It is a measure of how much of the total cropped area was sown with certified seeds in comparison to farm saved seeds. It represents the access of farmers to quality seed and directly proportional to productivity of farming.